Método de inducción matemática. El principio de inducción matemática. Ejemplos de soluciones

El texto de la obra se coloca sin imágenes ni fórmulas.
La versión completa del trabajo está disponible en la pestaña "Archivos de trabajo" en formato PDF.

Introducción

Este tema es relevante, ya que cada día las personas resuelven diversos problemas en los que aplican diferentes métodos de solución, pero hay tareas en las que no se puede prescindir del método de inducción matemática, y en tales casos el conocimiento en esta área será de gran utilidad.

Elegí este tema para la investigación porque en el plan de estudios de la escuela se dedica poco tiempo al método de inducción matemática, el estudiante aprende información superficial que lo ayudará a tener solo una idea general de este método, pero se requiere autodesarrollo para estudiar. esta teoría en profundidad. De hecho, será útil aprender más sobre este tema con más detalle, ya que amplía los horizontes de una persona y ayuda a resolver problemas complejos.

Objeto del trabajo:

Familiarizarse con el método de inducción matemática, sistematizar el conocimiento sobre este tema y aplicarlo en la resolución de problemas matemáticos y demostrar teoremas, fundamentar y mostrar claramente el valor práctico del método de inducción matemática como factor necesario para la resolución de problemas.

Tareas de trabajo:

    Analizar la literatura y resumir los conocimientos sobre este tema.

    Comprender el principio del método de inducción matemática.

    Explore la aplicación del método de inducción matemática a la resolución de problemas.

    Formular conclusiones y conclusiones sobre el trabajo realizado.

El cuerpo principal de la investigación

Historia de origen:

Solo a fines del siglo XIX hubo un estándar de requisitos de rigor lógico, que sigue siendo hasta el día de hoy dominante en el trabajo práctico de los matemáticos sobre el desarrollo de teorías matemáticas individuales.

La inducción es un procedimiento cognitivo mediante el cual se deriva un enunciado que los generaliza a partir de una comparación de los hechos disponibles.

En matemáticas, el papel de la inducción se debe en gran medida al hecho de que subyace a la axiomática elegida. Después de una larga práctica que demostró que un camino recto es siempre más corto que un camino curvo o roto, era natural formular un axioma: para tres puntos A, B y C cualesquiera, la desigualdad se mantiene.

La conciencia del método de inducción matemática como un método importante separado se remonta a Blaise Pascal y Gersonides, aunque Proclo y Euclides encontraron casos individuales de aplicación en la antigüedad. El nombre actual del método fue introducido por De Morgan en 1838.

El método de inducción matemática se puede comparar con el progreso: partimos de lo más bajo, como resultado del pensamiento lógico llegamos a lo más alto. El hombre siempre se ha esforzado por progresar, por la capacidad de desarrollar lógicamente su pensamiento, lo que significa que la naturaleza misma pretendía que pensara inductivamente.

Inducción y deducción

Se sabe que hay declaraciones tanto privadas como generales, y dos términos dados se basan en la transición de uno a otro.

Deducción (de Lat.deductio - retiro) - transición en el proceso de cognición de común conocimiento para privado y soltero... En la deducción, el conocimiento general sirve como punto de partida del razonamiento, y se supone que este conocimiento general está “listo”, existente. La peculiaridad de la deducción es que la verdad de sus premisas garantiza la verdad de la conclusión. Por lo tanto, la deducción tiene un tremendo poder de persuasión y se usa ampliamente no solo para probar teoremas en matemáticas, sino también donde se necesita un conocimiento confiable.

La inducción (del latín inductio - guía) es una transición en el proceso de cognición desde privado conocimiento para común En otras palabras, es un método de investigación, cognición asociada a la generalización de los resultados de observaciones y experimentos. Una característica de la inducción es su naturaleza probabilística, es decir. si las premisas iniciales son verdaderas, la conclusión de la inducción solo es probable que sea verdadera y en el resultado final puede resultar tanto verdadera como falsa.

Inducción completa e incompleta

La inferencia inductiva es una forma de pensamiento abstracto en la que el pensamiento se desarrolla desde un conocimiento de menor grado de generalidad hasta un conocimiento de mayor grado de generalidad, y la conclusión que surge de las premisas es predominantemente de naturaleza probabilística.

Durante mi investigación, descubrí que la inducción se divide en dos tipos: completa e incompleta.

La inducción completa se denomina inferencia, en la que se llega a una conclusión general sobre una clase de objetos sobre la base del estudio de todos los objetos de esta clase.

Por ejemplo, sea necesario establecer que todo número par natural n dentro de 6≤ n≤ 18 puede representarse como la suma de dos primos. Para hacer esto, tome todos esos números y escriba las expansiones correspondientes:

6=3+3; 8=5+3; 10=7+3; 12=7+5;14=7+7; 16=11+5; 18=13+5;

Estas igualdades muestran que cada uno de los números que nos interesan se representa de hecho como una suma de dos términos simples.

Considere el siguiente ejemplo: la secuencia yn = n 2 + n + 17; Escribamos los primeros cuatro términos: en 1 = 19; y 2 = 23; y 3 = 29; y 4 = 37; Entonces podemos suponer que toda la secuencia consta de números primos. Pero esto no es así, tome y 16 = 16 2 + 16 + 17 = 16 (16 + 1) + 17 = 17 * 17. Este es un número compuesto, lo que significa que nuestra suposición es incorrecta, por lo tanto, la inducción incompleta no conduce a conclusiones completamente confiables, pero nos permite formular una hipótesis, que en el futuro requiere una prueba o refutación matemática.

Método de inducción matemática

La inducción completa tiene un uso limitado en matemáticas. Muchos enunciados matemáticos interesantes cubren un número infinito de casos especiales y no podemos verificar todas estas situaciones, pero ¿cómo verificar un número infinito de casos? Este método fue propuesto por B. Pascal y J. Bernoulli, este es un método de inducción matemática, que se basa en principio de inducción matemática.

Si una oración A (n), dependiendo de un número natural n, es verdadera para n = 1 y del hecho de que es verdadera para n = k (donde k es cualquier número natural), se deduce que también es cierta para el siguiente número n = k +1, entonces el supuesto A (n) es verdadero para cualquier número natural n.

En algunos casos, es necesario probar la validez de un determinado enunciado no para todos los números naturales, sino solo para n> p, donde p es un número natural fijo. En este caso, el principio de inducción matemática se formula de la siguiente manera:

Si la oración А (n) es verdadera para n = py si А (k)  А (k + 1) para cualquier k> p, entonces la proposición А (n) es verdadera para cualquier n> p.

Algoritmo (consta de cuatro etapas):

1.base(mostramos que la afirmación que se está probando es verdadera para algunos de los casos especiales más simples ( NS = 1));

2.supuesto(asumimos que el enunciado se prueba por primera vez Para casos); 3 .paso(bajo este supuesto, probamos el enunciado del caso NS = Para + 1); 4.conclusión (en el enunciado es verdadero para todos los casos, es decir, para todos NS) .

Tenga en cuenta que no todos los problemas pueden resolverse mediante el método de inducción matemática, sino solo los problemas parametrizados por alguna variable. Esta variable se llama variable de inducción.

Aplicación del método de inducción matemática.

Aplicaremos toda esta teoría en la práctica y averiguaremos en qué problemas se utiliza este método.

Problemas para probar desigualdades.

Ejemplo 1. Demuestre la desigualdad de Bernoulli (1 + x) n≥1 + nx, x> -1, n € N.

1) Para n = 1, la desigualdad es válida, ya que 1 + x≥1 + x

2) Suponga que la desigualdad es verdadera para algunos n = k, es decir

(1 + x) k ≥1 + k x.

Multiplicando ambos lados de la desigualdad por un número positivo 1 + x, obtenemos

(1 + x) k + 1 ≥ (1 + kx) (1+ x) = 1 + (k + 1) x + kx 2

Teniendo en cuenta que kx 2 ≥0, llegamos a la desigualdad

(1 + x) k + 1 ≥1 + (k + 1) x.

Por tanto, del supuesto de que la desigualdad de Bernoulli es verdadera para n = k, se deduce que es verdadera para n = k + 1. Basado en el método de inducción matemática, se puede argumentar que la desigualdad de Bernoulli es válida para cualquier n € N.

Ejemplo 2. Demuestre que para cualquier número natural n> 1,.

Probémoslo usando el método de inducción matemática.

Denotamos el lado izquierdo de la desigualdad por.

1), por lo tanto, para n = 2, la desigualdad es válida.

2) Dejemos por unos k. Demostremos eso entonces y. Tenemos,.

Comparando y, tenemos, es decir ...

Para cualquier número natural k, el lado derecho de la última igualdad es positivo. Es por eso. Pero, por lo tanto, y. Hemos probado la desigualdad para n = k + 1, por lo tanto, por el método de inducción matemática, la desigualdad es válida para cualquier n> 1 natural.

Tareas de prueba de identidad.

Ejemplo 1. Demuestre que para cualquier n natural la siguiente igualdad es verdadera:

1 3 +2 3 +3 3 +… + n 3 = n 2 (n + 1) 2/4.

    Sea n = 1, luego X 1 = 1 3 = 1 2 (1 + 1) 2/4 = 1.

Vemos que para n = 1 el enunciado es verdadero.

2) Suponga que la igualdad es verdadera para n = kX k = k 2 (k + 1) 2/4.

3) Demostremos la verdad de esta afirmación para n = k + 1, es decir, X k + 1 = (k + 1) 2 (k + 2) 2/4. X k + 1 = 1 3 +2 3 +… + k 3 + (k + 1) 3 = k 2 (k + 1) 2/4 + (k + 1) 3 = (k 2 (k + 1) 2 +4 (k + 1) 3) / 4 = (k + 1) 2 (k 2 + 4k + 4) / 4 = (k + 1) 2 (k + 2) 2/4.

De la prueba anterior, está claro que la declaración es verdadera para n = k + 1, por lo tanto, la igualdad es verdadera para cualquier número natural n.

Ejemplo 2. Demuestre que para cualquier entero positivo n la igualdad

1) Comprobemos que esta identidad es verdadera para n = 1.; - Derecha.

2) Sea la identidad verdadera también para n = k, es decir

3) Probemos que esta identidad también es cierta para n = k + 1, es decir;

Porque la igualdad es verdadera para n = k y n = k + 1, entonces es verdadera para cualquier n natural.

Problemas de suma.

Ejemplo 1. Demuestre que 1 + 3 + 5 +… + (2n-1) = n 2.

Solución: 1) Tenemos n = 1 = 1 2. Por lo tanto, la afirmación es verdadera para n = 1, es decir A (1) es cierto.

2) Demostremos que A (k)  A (k + 1).

Sea k cualquier número natural y el enunciado sea verdadero para n = k, es decir, 1 + 3 + 5 +… + (2k-1) = k 2.

Demostremos que entonces el enunciado también es cierto para el siguiente número natural n = k + 1, es decir, qué

1 + 3 + 5 +… + (2k + 1) = (k + 1) 2.

De hecho, 1 + 3 + 5 +… + (2k-1) + (2k + 1) = k 2 + 2k + 1 = (k + 1) 2.

Entonces, A (k)  A (k + 1). Basado en el principio de inducción matemática, concluimos que el supuesto A (n) es verdadero para cualquier n N.

Ejemplo 2. Demuestre la fórmula, n es un número natural.

Solución: Para n = 1, ambos lados de la igualdad se vuelven uno y, por lo tanto, se satisface la primera condición del principio de inducción matemática.

Suponga que la fórmula es verdadera para n = k, es decir ...

Agregue esta igualdad a ambos lados y transforme el lado derecho. Entonces obtenemos

Por lo tanto, dado que la fórmula es verdadera para n = k, se deduce que también es cierta para n = k + 1, entonces esta afirmación es verdadera para cualquier número natural n.

Problemas de divisibilidad.

Ejemplo 1. Demuestre que (11 n + 2 +12 2n + 1) es divisible por 133 sin residuo.

Solución: 1) Sea n = 1, entonces

11 3 +12 3 = (11 + 12) (11 2-132 + 12 2) = 23 × 133.

(23 × 133) es divisible por 133 sin un residuo, por lo que para n = 1 el enunciado es verdadero;

2) Suponga que (11 k + 2 + 12 2k + 1) es divisible por 133 sin residuo.

3) Demostremos que en este caso

(11 k + 3 +12 2k + 3) es divisible por 133 sin residuo. De hecho, 11 k + 3 +12 2n + 3 = 11 × 11 k + 2 +

12 2 × 12 2k + 1 = 11 × 11 k + 2 + (11 + 133) × 12 2k + 1 = 11 (11 k + 2 +12 2k + 1) + 133 × 12 2k + 1.

La suma resultante es divisible por 133 sin residuo, ya que su primer término es divisible por 133 sin residuo por supuesto, y en el segundo de los factores es 133.

Entonces, A (k) → A (k + 1), entonces confiando en el método de inducción matemática, la declaración es verdadera para cualquier n natural.

Ejemplo 2. Demuestre que 3 3n-1 +2 4n-3 para un número natural arbitrario n es divisible por 11.

Solución: 1) Sea n = 1, luego X 1 = 3 3 - 1 + 2 4 - 3 = 3 2 + 2 1 = 11 es divisible por 11 sin un residuo. Por tanto, para n = 1 el enunciado es verdadero.

2) Suponga que para n = k

X k = 3 3k-1 +2 4k-3 es divisible por 11 sin residuo.

3) Demostremos que el enunciado es verdadero para n = k + 1.

X k + 1 = 3 3 (k + 1) -1 +2 4 (k + 1) -3 = 3 3k + 2 +2 4k + 1 = 3 3 * 3 3k-1 +2 4 * 2 4k-3 =

27 3 3k-1 + 16 * 2 4k-3 = (16 + 11) * 3 3k-1 + 16 * 2 4k-3 = 16 * 3 3k-1 +

11 * 3 3k-1 + 16 * 2 4k-3 = 16 (3 3k-1 + 2 4k-3) + 11 * 3 3k-1.

El primer término es divisible por 11 sin residuo, ya que 3 3k-1 +2 4k-3 es divisible por 11 por supuesto, el segundo es divisible por 11, porque uno de sus factores es 11. Esto significa que la suma es divisible por 11 sin un resto para cualquier n natural.

Tareas de la vida real.

Ejemplo 1. Demuestre que la suma Sn de los ángulos interiores de cualquier polígono convexo es ( NS- 2) π, donde NS- el número de lados de este polígono: Sn = ( NS- 2) π (1).

Esta afirmación no tiene sentido para todos los NS, pero solo para NS > 3, ya que el número mínimo de ángulos en un triángulo es 3.

1) Cuando NS= 3 nuestro enunciado toma la forma: S 3 = π. Pero la suma de los ángulos interiores de cualquier triángulo es realmente igual a π. Por lo tanto, en NS= 3 la fórmula (1) es correcta.

2) Sea esta fórmula verdadera para n = k, es decir, S k = (k- 2) π, donde k > 3. Demostremos que en este caso la fórmula también es válida: S k + 1 = (k- 1) π.

Sea A 1 A 2 ... A k A k + 1 - convexo arbitrario ( k+ 1) -gon (fig.338).

Conexión de los puntos A 1 y A k , obtenemos un convexo k-gon A 1 A 2 ... A k - 1 A k ... Obviamente, la suma de los ángulos ( k+ 1) -gon A 1 A 2 ... A k A k + 1 es igual a la suma de los ángulos k-gon A 1 A 2 ... A k más la suma de los ángulos del triángulo A 1 A k A k + 1. Pero la suma de los ángulos k-gon A 1 A 2 ... A k por supuesto es igual a ( k- 2) π, y la suma de los ángulos del triángulo A 1 A k A k + 1 es igual a π. Es por eso

S k + 1 = S k + π = ( k- 2) π + π = ( k- 1) π.

Entonces, ambas condiciones del principio de inducción matemática se satisfacen y, por lo tanto, la fórmula (1) es verdadera para cualquier NS > 3.

Ejemplo 2. Hay una escalera, todos los escalones son iguales. Se requiere indicar el número mínimo de posiciones que garantizarían la capacidad de "escalar" cualquier escalón por número.

Todos están de acuerdo en que debería haber una condición. Debemos poder subir el primer escalón. Además, deben poder subir del primer escalón al segundo. Luego, en el segundo, en el tercero, etc. al n-ésimo escalón. Por supuesto, en conjunto, las declaraciones "n" garantizan a nm que podremos llegar al n-ésimo paso.

Ahora miremos 2, 3,…., N posiciones y compárelas entre sí. Es fácil ver que todos tienen la misma estructura: si llegamos a k escalones, entonces podemos subir (k + 1) escalones. Por lo tanto, tal axioma para la validez de los enunciados que dependen de "n" se vuelve natural: si un enunciado A (n), en el que n es un número natural, se cumple para n = 1 y del hecho de que se cumple para n = k (donde k es cualquier número natural), se sigue que se cumple para n = k + 1, entonces el supuesto A (n) es válido para cualquier número natural n.

Solicitud

Tareas que utilizan el método de inducción matemática para el ingreso a las universidades.

Tenga en cuenta que al ingresar a las instituciones de educación superior, también hay problemas que se resuelven con este método. Considérelos con ejemplos específicos.

Ejemplo 1. Demuestra que cualquier natural NS igualdad justa

1) Cuando n = 1 obtenemos la igualdad correcta Sin.

2) Haciendo la hipótesis de inducción de que para n = k la igualdad es verdadera, considere la suma en el lado izquierdo de la igualdad, para n = k + 1;

3) Usando las fórmulas de fundición, transformamos la expresión:

Entonces, por el método de inducción matemática, la igualdad es verdadera para cualquier número natural n.

Ejemplo 2. Demuestre que para cualquier n natural el valor de la expresión 4n + 15n-1 es un múltiplo de 9.

1) Para n = 1: 2 2 + 15 - 1 = 18 - un múltiplo de 9 (desde 18: 9 = 2)

2) Deje que la igualdad se mantenga para n = k: 4 k + 15k-1 es múltiplo de 9.

3) Demostremos que la igualdad es válida para el siguiente número. n = k + 1

4 k + 1 +15 (k + 1) -1 = 4 k + 1 + 15k + 15-1 = 4.4 k + 60k-4-45k + 18 = 4 (4 k + 15k-1) -9 (5k- 2)

4 (4 k + 15k-1) - divisible por 9;

9 (5k-2) - divisible por 9;

Por lo tanto, toda la expresión 4 (4 k + 15k-1) -9 (5k-2) es un múltiplo de 9, que es lo que teníamos que demostrar.

Ejemplo 3. Demuestre que para cualquier número natural NS se cumple la condición: 1 ∙ 2 ∙ 3 ​​+ 2 ∙ 3 ​​∙ 4 +… + n (n + 1) (n + 2) =.

1) Comprobemos que esta fórmula es cierta para n = 1: Lado izquierdo = 1∙2∙3=6.

Parte derecha = . 6 = 6; cierto para n = 1.

2) Suponga que esta fórmula es verdadera para n = k:

1 ∙ 2 ∙ 3 ​​+ 2 ∙ 3 ​​∙ 4 +… + k (k + 1) (k + 2) =. S k =.

3) Demostremos que esta fórmula es válida para n = k + 1:

1 ∙ 2 ∙ 3 ​​+ 2 ∙ 3 ​​∙ 4 +… + (k + 1) (k + 2) (k + 3) =.

S k + 1 =.

Prueba:

Entonces, esta condición es verdadera en dos casos y demostró que es verdadera para n = k + 1, por tanto, es cierto para cualquier número natural NS.

Conclusión

Para resumir, en el proceso de investigación descubrí qué es la inducción, que es completa o incompleta, me familiaricé con el método de inducción matemática basado en el principio de inducción matemática, consideré muchos problemas usando este método.

También aprendí mucha información nueva, diferente a la que está incluida en el plan de estudios de la escuela. Al estudiar el método de inducción matemática, utilicé diversas publicaciones, recursos de Internet y también consulté con un maestro.

Producción: Habiendo generalizado y sistematizado los conocimientos sobre inducción matemática, me convencí de la necesidad de conocimiento sobre este tema en la realidad. Una cualidad positiva del método de inducción matemática es su amplia aplicación en la resolución de problemas: en el campo del álgebra, la geometría y las matemáticas reales. Además, este conocimiento aumenta el interés por las matemáticas como ciencia.

Estoy seguro de que las habilidades adquiridas en el curso del trabajo me ayudarán en el futuro.

Bibliografía

    Sominskiy I.S. Método de inducción matemática. Conferencias populares en matemáticas, número 3-M.: Nauka, 1974.

    L. I. Golovina, I. M. Yaglom. Inducción en geometría. - Fizmatgiz, 1961 .-- T.21 .-- 100 p. - (Conferencias populares en matemáticas).

    Dorofeev G.V., Potapov M.K., Rozov N.Kh. Un manual de matemáticas para quienes ingresan a las universidades (Preguntas seleccionadas de matemáticas elementales) - Publicación 5ta, revisada, 1976 - 638p.

    A. Shen. Inducción matemática. - MTsNMO, 2004 .-- 36 p.

    M.L. Galitsky, A.M. Goldman, L.I. Zvavich Colección de problemas de álgebra: libro de texto para grados 8-9. con profundización el estudio de las matemáticas 7ª ed.- M .: Educación, 2001. - 271 p.

    Ma-ka-ry-chev Yu.N., Min-duke N.G Capítulos completos del libro de texto escolar de ál-gebra de la novena clase. - M.: Pro-sves-shchenie, 2002.

    Wikipedia es la enciclopedia libre.

Introducción

Parte principal

1. Inducción completa e incompleta

2. El principio de inducción matemática

3. Método de inducción matemática

4. Solución de ejemplos

5. Igualdad

6. División de números

7. Desigualdades

Conclusión

Lista de literatura usada

Introducción

Toda la investigación matemática se basa en métodos deductivos e inductivos. El método deductivo de razonamiento es el razonamiento de lo general a lo particular, es decir. razonamiento, cuyo punto de partida es el resultado general, y el punto final es el resultado particular. La inducción se usa cuando se pasa de resultados particulares a generales, es decir, es lo opuesto al método deductivo.

El método de inducción matemática se puede comparar con el progreso. Comenzamos desde lo más bajo, como resultado del pensamiento lógico llegamos a lo más alto. El hombre siempre se ha esforzado por progresar, por la capacidad de desarrollar lógicamente su pensamiento, lo que significa que la naturaleza misma pretendía que pensara inductivamente.

Aunque el campo de aplicación del método de inducción matemática ha crecido, se le dedica poco tiempo en el currículo escolar. Bueno, diles que dos o tres lecciones traerán una persona útil, para la cual escuchará cinco palabras de teoría, resolverá cinco problemas primitivos y, como resultado, recibirá una A por no saber nada.

Pero es muy importante poder pensar de forma inductiva.

Parte principal

Según su significado original, la palabra "inducción" se aplica al razonamiento con la ayuda del cual se obtienen conclusiones generales, basadas en una serie de enunciados particulares. El método más simple de razonamiento de este tipo es la inducción completa. He aquí un ejemplo de este razonamiento.

Sea necesario establecer que todo número par natural n dentro de 4< n < 20 представимо в виде суммы двух простых чисел. Для этого возьмём все такие числа и выпишем соответствующие разложения:

4=2+2; 6=3+3; 8=5+3; 10=7+3; 12=7+5;

14=7+7; 16=11+5; 18=13+5; 20=13+7.

Estas nueve igualdades muestran que cada uno de los números que nos interesan se representa de hecho como una suma de dos términos simples.

Por tanto, la inducción completa significa que el enunciado general se prueba por separado en cada uno de un número finito de casos posibles.

A veces, el resultado general se puede predecir después de considerar no todos, sino un número suficientemente grande de casos especiales (la llamada inducción incompleta).

El resultado obtenido por inducción incompleta, sin embargo, sigue siendo solo una hipótesis hasta que sea probado por un razonamiento matemático exacto que cubra todos los casos especiales. En otras palabras, la inducción incompleta en matemáticas no se considera un método legítimo de demostración rigurosa, pero es un método poderoso para descubrir nuevas verdades.

Suponga, por ejemplo, que desea encontrar la suma de los primeros n números impares consecutivos. Consideremos casos especiales:

1+3+5+7+9=25=5 2

Después de considerar estos pocos casos especiales, se sugiere la siguiente conclusión general:

1 + 3 + 5 +… + (2n-1) = n 2

aquellos. la suma de los primeros n números impares consecutivos es n 2

Por supuesto, la observación realizada todavía no puede servir como prueba de la validez de la fórmula anterior.

La inducción completa tiene un uso limitado en matemáticas. Muchos enunciados matemáticos interesantes cubren un número infinito de casos especiales, pero no podemos verificar un número infinito de casos. La inducción incompleta a menudo conduce a resultados erróneos.

En muchos casos, la salida a este tipo de dificultad es recurrir a un método especial de razonamiento llamado método de inducción matemática. Es como sigue.

Suponga que necesita demostrar la validez de un determinado enunciado para cualquier número natural n (por ejemplo, necesita demostrar que la suma de los primeros n números impares es igual an 2). La verificación directa de esta declaración para cada valor de n es imposible, ya que el conjunto de números naturales es infinito. Para probar esta afirmación, primero verifique su validez para n = 1. Entonces se demuestra que para cualquier valor natural de k, la validez del enunciado considerado para n = k implica su validez también para n = k + 1.

Entonces, la declaración se considera probada para todo n. De hecho, la afirmación es cierta para n = 1. Pero también es cierto para el siguiente número n = 1 + 1 = 2. La validez del enunciado para n = 2 implica su validez para n = 2 +

1 = 3. Esto implica la validez de la declaración para n = 4, etc. Está claro que al final llegaremos a cualquier número natural n. Por tanto, el enunciado es verdadero para cualquier n.

Resumiendo lo dicho, formulamos el siguiente principio general.

El principio de inducción matemática.

Si la oración A ( norte ), dependiendo del número natural norte , cierto para norte = 1 y del hecho de que es cierto para n = k (dónde k -cualquier número natural), se deduce que también es cierto para el siguiente número n = k + 1 , entonces el supuesto A ( norte ) es cierto para cualquier número natural norte .

En algunos casos, es necesario probar la validez de un determinado enunciado no para todos los números naturales, sino solo para n> p, donde p es un número natural fijo. En este caso, el principio de inducción matemática se formula de la siguiente manera. Si la oración A ( norte ) es cierto para n = p y si A ( k ) Þ A( k + 1) para cualquiera k> p, luego la oración A ( norte) cierto para cualquiera n> p.

La demostración por el método de inducción matemática se lleva a cabo de la siguiente manera. Primero, la afirmación que se está probando se verifica para n = 1, es decir se establece la verdad del enunciado A (1). Esta parte de la prueba se llama base de inducción. Luego viene la parte de la demostración llamada paso de inducción. En esta parte, probamos la validez de la afirmación para n = k + 1 bajo el supuesto de que la afirmación es válida para n = k (la hipótesis de inducción), es decir, demuestre que A (k) ÞA (k + 1).

EJEMPLO 1

Demuestre que 1 + 3 + 5 +… + (2n-1) = n 2.

Solución: 1) Tenemos n = 1 = 1 2. Por eso,

la afirmación es verdadera para n = 1, es decir A (1) es cierto.

2) Demostremos que А (k) ÞA (k + 1).

Sea k cualquier número natural y el enunciado sea verdadero para n = k, es decir,

1 + 3 + 5 +… + (2k-1) = k 2.

Demostremos que entonces el enunciado también es cierto para el siguiente número natural n = k + 1, es decir, qué

1 + 3 + 5 +… + (2k + 1) = (k + 1) 2.

En efecto,

1 + 3 + 5 +… + (2k-1) + (2k + 1) = k 2 + 2k + 1 = (k + 1) 2.

Entonces, A (k) ÞA (k + 1). Con base en el principio de inducción matemática, concluimos que el supuesto A (n) es cierto para cualquier nÎN.

EJEMPLO 2

Pruebalo

1 + x + x 2 + x 3 + ... + x n = (x n + 1 -1) / (x-1), donde x¹1

Solución: 1) Para n = 1 obtenemos

1 + x = (x 2 -1) / (x-1) = (x-1) (x + 1) / (x-1) = x + 1

por tanto, para n = 1 la fórmula es correcta; A (1) es cierto.

2) Sea k cualquier número natural y sea la fórmula verdadera para n = k, es decir

1 + x + x 2 + x 3 + ... + x k = (x k + 1 -1) / (x-1).

Demostremos que entonces la igualdad

1 + x + x 2 + x 3 + ... + x k + x k + 1 = (x k + 2 -1) / (x-1).

En efecto

1 + x + x 2 + x 3 +… + x k + x k + 1 = (1 + x + x 2 + x 3 +… + x k) + x k + 1 =

= (x k + 1 -1) / (x-1) + x k + 1 = (x k + 2 -1) / (x-1).

Entonces, A (k) ÞA (k + 1). Con base en el principio de inducción matemática, concluimos que la fórmula es verdadera para cualquier número natural n.

EJEMPLO 3

Demuestre que el número de diagonales de un n-gon convexo es n (n-3) / 2.

Solución: 1) Para n = 3, el enunciado es


Y 3 es astuto, porque en un triángulo

 А 3 = 3 (3-3) / 2 = 0 diagonales;

A 2 A (3) es cierto.

2) Suponga que en cualquier

convexo k-gon tiene-

А 1 sy А k = k (k-3) / 2 diagonales.

А k Demostremos que entonces en el convexo

(k + 1) -número de gon

diagonales А k + 1 = (k + 1) (k-2) / 2.

Sea A 1 A 2 A 3… A k A k + 1 -convexo (k + 1) -ángulo. Dibuja una diagonal A 1 A k en ella. Para contar el número total de diagonales de este (k + 1) -gon, necesitas contar el número de diagonales en el k-gon A 1 A 2… A k, suma k-2 al número resultante, es decir se debe tener en cuenta el número de diagonales del (k + 1) -gon que sale del vértice А k + 1 y, además, la diagonal А 1 А k.

Por lo tanto,

 k + 1 =  k + (k-2) + 1 = k (k-3) / 2 + k-1 = (k + 1) (k-2) / 2.

Entonces, A (k) ÞA (k + 1). Debido al principio de inducción matemática, la afirmación es verdadera para cualquier n-gon convexo.

EJEMPLO 4

Demuestre que para cualquier n el siguiente enunciado es verdadero:

1 2 +2 2 +3 2 +… + n 2 = n (n + 1) (2n + 1) / 6.

Solución: 1) Sea n = 1, luego

X 1 = 1 2 = 1 (1 + 1) (2 + 1) / 6 = 1.

Por tanto, para n = 1 el enunciado es verdadero.

2) Suponga que n = k

X k = k 2 = k (k + 1) (2k + 1) / 6.

3) Considere esta afirmación para n = k + 1

X k + 1 = (k + 1) (k + 2) (2k + 3) / 6.

X k + 1 = 1 2 +2 2 +3 2 +… + k 2 + (k + 1) 2 = k (k + 1) (2k + 1) / 6 + + (k + 1) 2 = (k (k + 1) (2k + 1) +6 (k + 1) 2) / 6 = (k + 1) (k (2k + 1) +

6 (k + 1)) / 6 = (k + 1) (2k 2 + 7k + 6) / 6 = (k + 1) (2 (k + 3/2) (k +

2)) / 6 = (k + 1) (k + 2) (2k + 3) / 6.

Hemos probado la validez de la igualdad para n = k + 1, por lo tanto, en virtud del método de inducción matemática, el enunciado es verdadero para cualquier n natural.

EJEMPLO 5

Demuestre que para cualquier n natural la siguiente igualdad es verdadera:

1 3 +2 3 +3 3 +… + n 3 = n 2 (n + 1) 2/4.

Solución: 1) Sea n = 1.

Entonces X 1 = 1 3 = 1 2 (1 + 1) 2/4 = 1.

Vemos que para n = 1 el enunciado es verdadero.

2) Suponga que la igualdad es verdadera para n = k

Savelyeva Ekaterina

El artículo considera la aplicación del método de inducción matemática en la resolución de problemas de divisibilidad, a la suma de series. Se consideran ejemplos de aplicación del método de inducción matemática a la prueba de desigualdades y a la solución de problemas geométricos. El trabajo está ilustrado con una presentación.

Descargar:

Avance:

Ministerio de Ciencia y Educación de la Federación de Rusia

Institución educativa estatal

escuela secundaria número 618

En el curso: álgebra y los inicios del análisis

El tema del trabajo de diseño.

"El método de inducción matemática y su aplicación a la resolución de problemas"

Trabajo completado: Savelyeva E, clase 11B

Supervisor : Makarova T.P., profesora de matemáticas, GOU SOSH # 618

1. Introducción.

2. El método de inducción matemática para resolver problemas de divisibilidad.

3. Aplicación del método de inducción matemática a la suma de series.

4. Ejemplos de aplicación del método de inducción matemática a la demostración de desigualdades.

5. Aplicación del método de inducción matemática a la resolución de problemas geométricos.

6. Lista de literatura usada.

Introducción

Toda la investigación matemática se basa en métodos deductivos e inductivos. El método deductivo de razonamiento es el razonamiento de lo general a lo particular, es decir. razonamiento, cuyo punto de partida es el resultado general, y el punto final es el resultado particular. La inducción se usa cuando se pasa de resultados particulares a generales, es decir, es lo opuesto al método deductivo. El método de inducción matemática se puede comparar con el progreso-som. Comenzamos desde lo más bajo, como resultado del pensamiento lógico llegamos a lo más alto. El hombre siempre se ha esforzado por progresar, por la capacidad de desarrollar lógicamente su pensamiento, lo que significa que la naturaleza misma pretendía que pensara inductivamente. Aunque el campo de aplicación del método de inducción matemática ha crecido, no tiene mucho tiempo en el currículo escolar, y es muy importante poder pensar de manera inductiva. La aplicación de este principio en la resolución de problemas y la demostración de teoremas está a la par con la consideración en la práctica escolar y otros principios matemáticos: tercero excluido, inclusión-exclusión, Dirichlet, etc. La herramienta principal es el uso del método de inducción matemática. Hablando de la importancia de este método, A.N. Kolmogorov señaló que "la comprensión y la capacidad de aplicar el principio de inducción matemática es un buen criterio para la madurez, que es absolutamente necesaria para las matemáticas". El método de inducción en su sentido amplio consiste en la transición de observaciones particulares a un patrón general, universal o formulación general. En esta interpretación, el método es, por supuesto, la técnica principal para realizar investigaciones en cualquier ciencia natural experimental.

actividades humanas. El método (principio) de inducción matemática en su forma más simple se usa cuando necesitas probar alguna declaración para todos los números naturales.

Problema 1. En su artículo "Cómo me convertí en matemático", A.N. Kolmogorov escribe: "Aprendí la alegría de un" descubrimiento "matemático temprano, notando a la edad de cinco o seis años la regularidad

1 =1 2 ,

1 + 3 = 2 2 ,

1 + 3 + 5 = Ç 2,

1 + 3 + 5 + 7 = 4 2 y así sucesivamente.

La escuela publicó la revista "Spring Swallows". Publicó mi descubrimiento ... "

No sabemos qué tipo de prueba se dio en esta revista, pero todo comenzó con observaciones privadas. La misma hipótesis, que probablemente surgió después del descubrimiento de estas igualdades parciales, es que la fórmula

1 + 3 + 5 + ... + (2n - 1) = n 2

cierto para cualquier número dado n = 1, 2, 3, ...

Para probar esta hipótesis, basta con establecer dos hechos. Primero, por n = 1 (e incluso para n = 2, 3, 4) la afirmación requerida es verdadera. En segundo lugar, suponga que el enunciado es verdadero para n = k, y asegúrese de que sea cierto para n = k + 1:

1 + 3 + 5 + ... + (2k - 1) + (2k + 1) = (1 + 3 + 5 + ... + (2k - 1)) + (2k + 1) = 2 + (2k + 1) = (k + I) 2.

Por lo tanto, la afirmación que se está probando es verdadera para todos los valores. n: para n = 1 es verdad (esto se verifica), y en virtud del segundo hecho - por n = 2, de donde para n = 3 (en virtud del mismo, segundo hecho), etc.

Problema 2. Considere todas las fracciones ordinarias posibles con el numerador 1 y cualquier (posición entera

denominador: Demuestre que para cualquier n> 3 se puede representar como una unidad como una suma NS diferentes fracciones de este tipo.

Solución, Primero revisemos esta declaración para n = 3; tenemos:

Por tanto, se cumple la afirmación básica.

Supongamos ahora que el enunciado que nos interesa es verdadero para algún número Para, y demuestre que también es cierto para el siguiente número Para + 1. En otras palabras, suponga que hay una representación

donde k términos y todos los denominadores son diferentes. Demostremos que entonces es posible obtener una representación de la unidad en forma de suma a partir de Para + 1 fracciones del tipo deseado. Supondremos que disminuyen las fracciones, es decir, los denominadores (en la representación de la unidad por la suma Para términos) aumentan de izquierda a derecha para que T Es el mayor de los denominadores. Obtendremos la representación que necesitamos en forma de suma(Para + 1) la fracción, si dividimos una fracción, por ejemplo la última, en dos. Esto se puede hacer desde

Y por lo tanto

Además, todas las fracciones se mantuvieron diferentes, ya que T fue el mayor denominador y m + 1> m, y

t (t + 1)> t.

Así, hemos establecido:

  1. para n = 3 esta afirmación es verdadera;
  1. si la declaración de interés para nosotros es verdadera para Para,
    entonces también es cierto para k + 1.

Sobre esta base, podemos afirmar que el enunciado en consideración es verdadero para todos los números naturales, comenzando con tres. Además, la prueba anterior también implica un algoritmo para encontrar la partición requerida de la unidad. (¿Qué tipo de algoritmo es este? Imagine el número 1 como una suma de 4, 5, 7 términos usted mismo).

Al resolver las dos tareas anteriores, se dieron dos pasos. El primer paso se llama base inducción, el segundo -transición inductivao por paso de inducción. El segundo paso es el más importante e incluye un supuesto (el enunciado es verdadero para n = k) y la conclusión (la afirmación es verdadera para n = k + 1). El propio parámetro n se llama por el parámetro de inducción.Este esquema lógico (técnica), que nos permite concluir que el enunciado en consideración es verdadero para todos los números naturales (o para todos, comenzando por alguno), ya que tanto la base como la transición son verdaderas, se llamael principio de inducción matemática, en el cual y se basa el método de inducción matemática.El término "inducción" en sí mismo proviene de la palabra latina induktio (orientación), que significa la transición de un conocimiento único sobre objetos individuales de una clase determinada a una conclusión general sobre todos los objetos de una clase determinada, que es uno de los principales métodos de cognición.

El principio de inducción matemática, precisamente en la forma familiar de dos pasos, apareció por primera vez en 1654 en el Tratado de Blaise Pascal sobre el triángulo aritmético, en el que se demostró un método simple para calcular el número de combinaciones (coeficientes binomiales) por inducción. D. Polya en el libro cita a B. Pascal con cambios menores entre corchetes:

“A pesar de que la oración bajo consideración [una fórmula explícita para coeficientes binomiales] contiene innumerables casos especiales, daré una prueba muy breve basada en dos lemas.

El primer lema afirma que la suposición es verdadera para la base; esto es obvio. [A NS = 1 la fórmula explícita es válida ...]

El segundo lema afirma lo siguiente: si nuestra suposición es verdadera para una base arbitraria [para una n arbitraria], entonces también será cierta para la siguiente base [para n + 1].

Estos dos lemas implican necesariamente la validez de la proposición para todos los valores. NS. De hecho, en virtud del primer lema, es válido para NS = 1; por tanto, en virtud del segundo lema, es válido para NS = 2; por tanto, de nuevo en virtud del segundo lema, es válido para n = 3 y así sucesivamente hasta el infinito ".

Problema 3. El rompecabezas "Torres de Hanoi" consta de tres varillas. En una de las varillas hay una pirámide (Fig.1), que consta de varios anillos de diferentes diámetros, disminuyendo de abajo hacia arriba.

Figura 1

Esta pirámide debe moverse a una de las otras varillas, transfiriendo solo un anillo a la vez y no colocando el anillo más grande en el más pequeño. Se puede hacer esto?

Solución. Entonces, debemos responder a la pregunta: ¿es posible mover la pirámide que consta de NS anillos de diferentes diámetros, de una caña a otra, observando las reglas del juego? Ahora el problema está, como dicen, parametrizado por nosotros (hemos introducido un número natural NS), y puede resolverse mediante el método de inducción matemática.

  1. Base de inducción. Para n = 1, todo está claro, ya que una pirámide de un anillo obviamente se puede mover a cualquier barra.
  2. Paso de inducción. Supongamos que podemos mover cualquier pirámide con el número de anillos n = k.
    Demostremos que entonces podemos mover la pirámide con n = k + 1.

Pirámide de a anillos que se encuentran en el más grande(Para + 1) th ring, podemos, según el supuesto, movernos a cualquier otra varilla. Vamos a hacerlo. Estacionario(Para + 1) -ésimo anillo no interferirá con el algoritmo de movimiento, ya que es el más grande. Después de mudarse Para anillos, mueve este más grande(Para + 1) aro en la varilla restante. Y luego aplicamos nuevamente el algoritmo de movimiento que conocemos de la hipótesis inductiva Para anillos, y muévalos a la varilla con el(Para + 1) th anillo. Por lo tanto, si somos capaces de mover las pirámides con Para anillos, entonces podemos mover las pirámides y con Para + 1 anillos. Por lo tanto, de acuerdo con el principio de inducción matemática, siempre es posible mover la pirámide, que consiste en n anillos, donde n> 1.

El método de inducción matemática para resolver problemas de divisibilidad.

Utilizando el método de inducción matemática, se pueden probar varias afirmaciones sobre la divisibilidad de los números naturales.

Problema 4 ... Si n es un número natural, entonces el número es par.

Para n = 1 nuestra afirmación es verdadera: - un número par. Suponga que es un número par. Dado que 2k es un número par, entonces también es par. Entonces, la paridad se demuestra para n = 1, la paridad se deduce de la paridad, es decir, incluso para todos los valores naturales de n.

Problema 3. Demuestre que el número Ç 3 + 3 - 26n - 27 con natural arbitrario n es divisible por 26 2 sin resto.

Solución. Primero, probamos por inducción una afirmación auxiliar de que 3 3n + 3 - 1 es divisible por 26 sin resto en n> 0.

  1. Base de inducción. Para n = 0 tenemos: Ç 3 - 1 = 26 - divisible por 26.

Paso de inducción. Suponga 3 3n + 3 - 1 dividido por 26 cuando n = k, y Demostremos que en este caso el enunciado será verdadero para n = k + 1. Dado que 3

luego de la hipótesis inductiva concluimos que el número 3 3k + 6-1 es divisible por 26.

Ahora demostremos el enunciado formulado en el enunciado del problema. Y nuevamente por inducción.

  1. Base de inducción. Obviamente, para n = 1 afirmación es verdadera: ya que 3 3+3 - 26 - 27 = 676 = 26 2 .
  2. Paso de inducción. Supongamos que para n = k
    expresión 3 3k + 3 - 26k - 27 dividido por 26 2 sin resto, y demostrar que el enunciado es verdadero para n = k + 1,
    ese es el numero

divisible por 26 2 sin un resto. En la última suma, ambos términos son divisibles por 26 sin dejar residuo 2 ... Primero, porque hemos probado que la expresión entre paréntesis es divisible por 26; el segundo es por la hipótesis de inducción. En virtud del principio de inducción matemática, el enunciado requerido está completamente probado.

Aplicación del método de inducción matemática a la suma de series.

Tarea 5. Demuestra la fórmula

N es un número natural.

Solución.

Para n = 1, ambos lados de la igualdad se vuelven uno y, por lo tanto, se satisface la primera condición del principio de inducción matemática.

Suponga que la fórmula es verdadera para n = k, es decir

Agregue esta igualdad a ambos lados y transforme el lado derecho. Entonces obtenemos

Por tanto, dado que la fórmula es verdadera para n = k, se deduce que también es cierta para n = k + 1. Esta afirmación es cierta para cualquier valor natural de k. Por tanto, también se satisface la segunda condición del principio de inducción matemática. La fórmula está probada.

Tarea 6. Hay dos números escritos en la pizarra: 1.1. Habiendo ingresado su suma entre los números, obtenemos los números 1, 2, 1. Repitiendo esta operación nuevamente, obtenemos los números 1, 3, 2, 3, 1. Después de tres operaciones, habrá números 1, 4, 3 , 5, 2, 5, 3, 4, 1. ¿Cuál será la suma de todos los números en el tablero después 100 operaciones?

Solución. Completa los 100 las operaciones llevarían mucho tiempo y consumirían mucho tiempo. Por lo tanto, debemos intentar encontrar alguna fórmula general para la suma S números después de n operaciones. Veamos la tabla:

¿Has notado algún patrón aquí? Si no es así, puede dar un paso más: después de cuatro operaciones habrá números

1, 5, 4, 7, 3, 8, 5, 7, 2, 7, 5, 8, 3, 7, 4, 5, 1,

cuya suma S 4 es 82.

De hecho, no puede escribir los números, pero decir inmediatamente cómo cambiará la cantidad después de agregar nuevos números. Sea la suma 5. ¿En qué se convertirá cuando se agreguen nuevos números? Dividamos cada nuevo número en la suma de los dos anteriores. Por ejemplo, de 1, 3, 2, 3, 1 vamos a 1,

1 + 3, 3, 3 + 2, 2, 2 + 3, 3, 3 + 1, 1.

Es decir, cada número anterior (excepto las dos unidades extremas) ahora se incluye en la suma tres veces, por lo que la nueva suma es igual a 3S - 2 (reste 2 para tener en cuenta las unidades faltantes). Por lo tanto S 5 = 3S 4 - 2 = 244, y en general

Cual es la formula general? Si no fuera por la resta de dos unidades, entonces cada vez la suma se triplicaría, como en potencias de un triple (1, 3, 9, 27, 81, 243, ...). Y nuestros números, como puede ver ahora, son uno más. Por tanto, se puede suponer que

Intentemos ahora probar esto por inducción.

Base de inducción. Ver tabla (para n = 0, 1, 2, 3).

Paso de inducción. Pretendamos que

Entonces probaremos que S k + 1 = Z k + 1 + 1.

En realidad,

Entonces, nuestra fórmula está probada. Se puede ver en él que después de cien operaciones, la suma de todos los números en el tablero será igual a З 100 + 1.

Considere un gran ejemplo de la aplicación del principio de inducción matemática, en el que primero debe introducir dos parámetros naturales y luego la inducción en su suma.

Tarea 7. Demuestre que si= 2, x 2 = 3 y por cada natural n> 3 la relación se mantiene

x n = Zx n - 1 - 2x n - 2,

luego

2 p - 1 + 1, n = 1, 2, 3, ...

Solución. Tenga en cuenta que en este problema la secuencia original de números(x n) está determinada por inducción, ya que los miembros de nuestra secuencia, además de los dos primeros, se dan inductivamente, es decir, a través de los anteriores. Así es como se llaman las secuencias dadas recurrente, y en nuestro caso esta secuencia se determina (especificando los dos primeros de sus miembros) de forma única.

Base de inducción. Consiste en comprobar dos declaraciones: para n = 1 y n = 2.B en ambos casos, la afirmación es verdadera por condición.

Paso de inducción. Supongamos que para n = k - 1 y n = k la declaración se cumple, es decir

Entonces demostremos la validez del enunciado para n = k + 1. Tenemos:

x 1 = 3 (2 + 1) - 2 (2 + 1) = 2 + 1, según sea necesario.

Problema 8. Demuestre que cualquier número natural puede representarse como la suma de varios miembros diferentes de la secuencia recurrente de números de Fibonacci:

para k> 2.

Solución. Deje n - número natural. Realizaremos la inducción en NS.

Base de inducción. Para n = La declaración 1 es verdadera, ya que la unidad en sí misma es un número de Fibonacci.

Paso de inducción. Suponga que todos los números naturales menores que algún número NS, se puede representar como la suma de varios miembros diferentes de la secuencia de Fibonacci. Encuentra el número de Fibonacci más grande F t, no superior NS; por tanto, F m n y F m +1> n.

En la medida en

Según la hipótesis de inducción, el número n- F t puede representarse como una suma de 5 de varios miembros diferentes de la secuencia de Fibonacci, y de la última desigualdad se deduce que todos los miembros de la secuencia de Fibonacci involucrados en la suma de 8 son menos F t. Por tanto, la expansión del número n = 8 + F t satisface la condición del problema.

Ejemplos de la aplicación del método de inducción matemática a la prueba de desigualdades.

Problema 9. (La desigualdad de Bernoulli).Demuestre eso para x> -1, x 0 y para el entero n> 2 la desigualdad

(1 + x) n> 1 + xn.

Solución. La prueba se volverá a realizar por inducción.

1. Base de inducción. Verifiquemos la desigualdad para n = 2. De hecho,

(1 + x) 2 = 1 + 2x + x 2> 1 + 2x.

2. Paso de inducción. Supongamos que para el número n = k la afirmación es verdadera, es decir

(1 + x) k> 1 + xk,

Donde k> 2. Lo demostramos para n = k + 1. Tenemos: (1 + x) k + 1 = (1 + x) k (1 + x)> (1 + kx) (1 + x) =

1 + (k + 1) x + kx 2> 1 + (k + 1) x.

Entonces, basado en el principio de inducción matemática, se puede argumentar que la desigualdad de Bernoulli es válida para cualquier n> 2.

No siempre en las condiciones de los problemas resueltos mediante el método de inducción matemática, se formula claramente una ley general que debe demostrarse. A veces es necesario, al observar casos particulares, primero descubrir (adivinar) a qué ley general conducen, y solo entonces probar la hipótesis planteada por el método de inducción matemática. Además, se puede enmascarar la variable de inducción, y antes de resolver el problema, es necesario determinar con qué parámetro se realizará la inducción. Considere las siguientes tareas como ejemplos.

Problema 10. Demuestre que

con cualquier natural n> 1.

Solución, Intentemos demostrar esta desigualdad mediante inducción matemática.

La base de inducción se verifica fácilmente: 1+

Por la hipótesis de la inducción

y nos queda demostrar que

Usando la hipótesis inductiva, afirmaremos que

Si bien esta igualdad es realmente cierta, no nos da una solución al problema.

Intentemos probar una afirmación más fuerte que la requerida en el problema original. Es decir, demostremos que

Puede parecer que probar esta afirmación por inducción es una tarea inútil.

Sin embargo, para n = 1 tenemos: la afirmación es verdadera. Para justificar el paso inductivo, suponga que

y luego probar eso

En realidad,

Por lo tanto, hemos demostrado una afirmación más fuerte, de la cual se sigue inmediatamente la afirmación contenida en el enunciado del problema.

Es instructivo aquí que, aunque tuvimos que probar un enunciado más fuerte de lo que se requiere en el problema, podríamos haber utilizado un supuesto más sólido en el paso inductivo. Esto explica por qué una aplicación sencilla del principio de inducción matemática no siempre conduce a la meta.

La situación que surgió al resolver el problema se denominóla paradoja del inventor.La paradoja en sí es que se pueden implementar planes más complejos con mayor éxito si se basan en una comprensión más profunda de la esencia del asunto.

Problema 11. Demuestre que 2 m + n - 2 m con cualquier natural tipo de.

Solución. Tenemos dos parámetros aquí. Por lo tanto, puede intentar realizar el llamadodoble inducción(inducción dentro de la inducción).

Realizaremos razonamientos inductivos sobre NS.

1. Base de inducción en la pág. Para n = 1 es necesario comprobar que 2 t ~ 1> t. Para probar esta desigualdad, usamos la inducción en T.

a) Base de inducción en m. Cuando m = 1 corriendo
igualdad, que es permisible.

B) El paso de inducción en v.Supongamos que para t = k la afirmación es verdadera, es decir 2 k ~ 1> k. Entonces antes
creemos que la afirmación será cierta también para
m = k + 1.
Tenemos:

con natural c.

Por tanto, la desigualdad 2 realizado con cualquier natural T.

2. La inducción paso por p.Elijamos y arreglemos un número natural. T. Supongamos que para n = yo la afirmación es verdadera (por un tiempo fijo m), es decir, 2 m +1 ~ 2> m1, y demostrar que entonces la afirmación también es válida para n = l + 1.
Tenemos:

con cualquier natural tipo de.

Por lo tanto, basado en el principio de inducción matemática (por NS) el enunciado del problema es verdadero para cualquier NS y para cualquier fijo T. Por tanto, esta desigualdad es válida para cualquier tipo de.

Problema 12. Sean m, n y k Son números naturales y m> n. ¿Cuál de los dos números es mayor?

En cada expresión Para signos de raíz cuadrada, myn se alternan.

Solución. Probemos primero una determinada afirmación auxiliar.

Lema. Con cualquier natural myn (m> n) y no negativo (no necesariamente completo) NS la desigualdad es verdadera

Prueba. Considere la desigualdad

Esta desigualdad es cierta, ya que ambos factores del lado izquierdo son positivos. Ampliando los corchetes y transformando, obtenemos:

Tomando la raíz cuadrada de ambos lados de la última desigualdad, obtenemos la afirmación del lema. Entonces, el lema está probado.

Ahora pasemos a resolver el problema. Denotemos el primero de estos números por a, y el segundo - a través B k. Demostremos que un con cualquier natural Para. La demostración se realizará por el método de inducción matemática por separado para pares e impares. Para.

Base de inducción. Para k = 1 tenemos la desigualdad

y [t> y / n , que es válido por el hecho de que m> n. Para k = 2 el resultado requerido se obtiene del lema probado sustituyendo x = 0.

Paso de inducción. Supongamos que para algunos k desigualdad a> b k justa. Demostremos que

A partir del supuesto de inducción y la monotonicidad de la raíz cuadrada, tenemos:

Por otro lado, del lema probado se sigue que

Combinando las dos últimas desigualdades, obtenemos:

Según el principio de inducción matemática, la afirmación está probada.

Problema 13. (Desigualdad de Cauchy.)Demuestre que para cualquier número positivo ..., un la desigualdad es verdadera

Solución. Para n = 2, la desigualdad

la media aritmética y la media geométrica (para dos números) se considerarán conocidas. Permitir n = 2, k = 1, 2, 3, ... y primero usamos la inducción en Para. La base de esta inducción tiene lugar suponiendo ahora que ya se ha establecido la desigualdad requerida para n = 2, lo probamos por NS = 2. Tenemos (usando la desigualdad para dos números):

Por tanto, según la hipótesis de inducción

Así, por inducción sobre k, hemos probado la desigualdad para todos n 9 que son una potencia de dos.

Para probar la desigualdad para otros valores. NS usamos "inducción descendente", es decir, demostramos que si la desigualdad se cumple para arbitrarias no negativas NS números, entonces también es cierto para(NS - 1) número. Para verificar esto, tenga en cuenta que, por la suposición hecha, para NS números, la desigualdad

es decir, a r + a 2 + ... + a n _ x> (n - 1) A. Dividiendo ambas partes en NS - 1, obtenemos la desigualdad requerida.

Entonces, primero establecimos que la desigualdad es válida para un número infinito de valores posibles NS, y luego demostró que si la desigualdad es válida para NS números, entonces también es cierto para(NS - 1) números. De esto ahora concluimos que la desigualdad de Coty es válida para un conjunto de NS cualquier número no negativo para cualquier n = 2, 3, 4, ...

Problema 14. (D. Uspensky.) Para cualquier triángulo ABC con ángulos = CAB, = CBA son conmensurables, las desigualdades

Solución. Los ángulos y son conmensurables, y esto (por definición) significa que estos ángulos tienen una medida común, para la cual = p, = (p, q son números coprimos naturales).

Usaremos el método de inducción matemática y lo realizaremos sobre la suma n = p + q números coprimos naturales.

Base de inducción. Para p + q = 2 tenemos: p = 1 y q = 1. Entonces el triángulo ABC es isósceles, y las desigualdades necesarias son obvias: se siguen de la desigualdad del triángulo

Paso de inducción. Supongamos ahora que se han establecido las desigualdades requeridas para p + q = 2,3, ..., k - 1, donde k> 2. Demostremos que las desigualdades también son válidas para p + q = k.

Deja que ABC - un triángulo dado, en el que> 2. Luego los lados AC y BC no puede ser igual: dejaАС> ВС. Ahora construimos, como en la Figura 2, un triángulo isósceles A B C; tenemos:

AC = DC y AD = AB + BD, por lo tanto

2AC> AB + BD (1)

Considere ahora un triánguloВDC, cuyos ángulos también son comparables:

DCB = (q - p), BDC = p.

Arroz. 2

La suposición inductiva se cumple para este triángulo, y por lo tanto

(2)

Sumando (1) y (2), tenemos:

2AC + BD>

y por lo tanto

Del mismo triangulo VBS por la hipótesis de inducción, concluimos que

Teniendo en cuenta la desigualdad anterior, concluimos que

Así, se obtiene la transición inductiva y el planteamiento del problema se deriva del principio de inducción matemática.

Comentario. El enunciado del problema sigue siendo válido incluso en el caso de que los ángulos ayp no sean conmensurables. En el caso general, la consideración debe basarse en otro principio matemático importante: el principio de continuidad.

Problema 15. Varias líneas rectas dividen el avión en partes. Demuestra que puedes pintar estas partes de blanco

y negro de tal manera que las partes adyacentes que tienen un segmento de borde común sean de diferentes colores (como en la Figura 3 con n = 4).

imagen 3

Solución. Usamos la inducción en el número de líneas. Entonces deja NS - el número de líneas rectas que dividen nuestro avión en partes, n> 1.

Base de inducción. Si la linea recta esta sola(NS = 1), luego divide el plano en dos semiplanos, uno de los cuales puede ser de color blanco y el otro negro, y el enunciado del problema es verdadero.

Paso de inducción. Para que la prueba de la transición inductiva sea más clara, considere el proceso de agregar una nueva línea. Si sacamos la segunda recta(NS= 2), luego obtenemos cuatro partes que se pueden colorear de la manera deseada pintando las esquinas opuestas del mismo color. Veamos qué pasa si sacamos la tercera recta. Dividirá algunas de las partes "antiguas" y aparecerán nuevas secciones del borde, en ambos lados de los cuales el color es el mismo (Fig. 4).

Arroz. 4

Procedamos de la siguiente manera:un ladocambie los colores de la nueva línea recta - haga blanco negro y viceversa; en este caso, no repintamos las partes que se encuentran al otro lado de esta línea recta (Fig. 5). Entonces esta nueva coloración satisfará los requisitos necesarios: por un lado, la línea recta, ya estaba alternando (pero con diferentes colores), y por otro lado, era necesaria. Para que las partes que tienen un borde común perteneciente a la línea recta dibujada se pinten en diferentes colores, repintamos las partes solo en un lado de esta línea recta dibujada.

Figura 5

Probemos ahora la transición inductiva. Supongamos que para algunosn = kel enunciado del problema es verdadero, es decir, todas las partes del plano en el que está dividido por estosPararecta, se puede pintar en blanco y negro para que las partes adyacentes sean de diferentes colores. Demostremos que entonces existe tal coloración paraNS= Para+ 1 líneas rectas. Procedemos de manera similar al caso de pasar de dos rectas a tres. Vamos a gastar en el aviónParadirecto. Luego, según la hipótesis de inducción, el "mapa" resultante se puede colorear según sea necesario. Gastemos ahora(Para+ 1) la línea recta y en un lado de la misma cambiamos los colores a opuestos. Y ahora(Para+ 1) -a línea recta en todas partes separa secciones de diferentes colores, mientras que las partes "antiguas", como ya hemos visto, permanecen correctamente coloreadas. Según el principio de inducción matemática, el problema está resuelto.

Tarea16. Al borde del desierto hay una gran cantidad de gasolina y un automóvil que puede viajar 50 kilómetros cuando está completamente repostado. Hay botes ilimitados en los que puede verter gasolina desde el tanque de gasolina del automóvil y dejarlo para almacenar en cualquier lugar del desierto. Demuestre que un automóvil puede viajar cualquier distancia entera superior a 50 kilómetros. No está permitido llevar latas de gasolina, las latas vacías se pueden transportar en cualquier cantidad.

Solución.Intentaremos probarlo por inducción enNS,que el coche puede arrancarNSkilómetros del borde del desierto. ANS= 50 se sabe. Queda por realizar el paso de inducción y explicar cómo llegar.n = k+ 1 kilómetro si se sabe quen = kkilómetros que puede conducir.

Sin embargo, aquí nos enfrentamos a una dificultad: después de haber pasadoParakilómetros, la gasolina puede no ser suficiente para el viaje de regreso (sin mencionar el almacenamiento). Y en este caso, la salida es fortalecer la afirmación que se está probando (la paradoja del inventor). Demostraremos que no solo puedes conducirNSkilómetros, pero también hacen un suministro arbitrariamente grande de gasolina en un punto a una distanciaNSkilómetros desde el borde del desierto, llegando a este punto después del final del transporte.

Base de inducción.Sea la unidad de gasolina la cantidad de gasolina necesaria para viajar un kilómetro. Luego, un viaje de 1 kilómetro y vuelta requiere dos unidades de gasolina, por lo que podemos dejar 48 unidades de gasolina almacenadas a un kilómetro del borde y regresar por una nueva porción. Por lo tanto, para varios vuelos al almacenamiento, podemos hacer un stock de un tamaño arbitrario que necesitemos. Al mismo tiempo, para crear 48 unidades de stock, consumimos 50 unidades de gasolina.

Paso de inducción.Supongamos que a distanciaNS= Paradesde el borde del desierto, puede abastecerse de cualquier cantidad de gasolina. Demostremos que entonces es posible crear un almacenamiento a distancian = k+ 1 km con cualquier suministro predeterminado de gasolina y estar en este almacenamiento al final del transporte. Ya que en el puntoNS= Parahay un suministro ilimitado de gasolina, luego (según la base de inducción) podemos hacer varios viajes al punton = k+ 1 hacer en el puntoNS= Para4 - 1 stock de cualquier tamaño que necesite.

La verdad de un enunciado más general que en la condición del problema se sigue ahora del principio de inducción matemática.

Conclusión

En particular, después de haber estudiado el método de inducción matemática, mejoré mis conocimientos en esta área de las matemáticas y también aprendí cómo resolver problemas que antes estaban fuera de mi alcance.

Básicamente, estas eran tareas lógicas y entretenidas, es decir, solo aquellos que aumentan el interés en las matemáticas como ciencia. Resolver este tipo de problemas se convierte en una actividad entretenida y puede atraer cada vez a más personas curiosas a los laberintos matemáticos. En mi opinión, esta es la base de cualquier ciencia.

Continuando con el estudio del método de inducción matemática, intentaré aprender a aplicarlo no solo en matemáticas, sino también en la resolución de problemas de física, química y la vida misma.

Literatura

1. INDUCCIÓN Vulenkin. Combinatoria. Una guía para profesores. M., Iluminación,

1976.-48 p.

2 Golovina L.I., Yaglom I.M. Inducción en geometría. - M.: Gosud. publicado. carta. - 1956 - S. I00. Un manual de matemáticas para aspirantes a la universidad / Ed. Yakovleva G.N. La ciencia. -1981. - S.47-51.

3 Golovina L.I., Yaglom I.M. Inducción en geometría. -
M .: Nauka, 1961. - (Charlas populares sobre matemáticas.)

4. I.T. Demidov, A.N. Kolmogorov, S.I.Schwarzburg, O.S.Ivashev-Musatov, B.E. Weitz. Libro de texto / "Educación" 1975.

5.R. Courant, G. Robbins "¿Qué son las matemáticas?" Capítulo 1, § 2

6. Popa D. Matemáticas y razonamiento plausible. - M: Ciencia, 1975.

7. Popa D. Descubrimiento matemático. - M .: Nauka, 1976.

8.Rubanov I.S. Cómo enseñar el método de inducción matemática / escuela de matemáticas. - Nl. - 1996. - P.14-20.

9. Sominskiy I.S., Golovina L.I., Yaglom I.M. Sobre el método de inducción matemática. - M .: Nauka, 1977. - (Charlas populares sobre matemáticas.)

10. Solominsky I.S. Método de inducción matemática. - M.: Ciencia.

63s.

11. Solominsky I.S., Golovina L.I., Yaglom I.M. Sobre la inducción matemática. - M .: Ciencia. - 1967. - P.7-59.

12.httr: //sh.wikiiredia.org/wiki

13.htt12: / /www.refeshtcollectiop.ru/40 124.html

El verdadero conocimiento en todo momento se basó en el establecimiento de un patrón y la prueba de su veracidad en determinadas circunstancias. Durante un período tan largo de existencia del razonamiento lógico, se dieron formulaciones de las reglas, y Aristóteles incluso hizo una lista de "razonamiento correcto". Históricamente, se acostumbra dividir todas las inferencias en dos tipos: desde lo concreto hasta lo múltiple (inducción) y viceversa (deducción). Cabe señalar que los tipos de evidencia de particular a general y de general a particular existen solo en interrelación y no pueden ser intercambiables.

Inducción en matemáticas

El término "inducción" (inducción) tiene raíces latinas y se traduce literalmente como "guía". En un examen más detenido, se puede distinguir la estructura de la palabra, a saber, el prefijo latino - in (denota acción dirigida hacia adentro o estar adentro) y -ducción - introducción. Cabe señalar que hay dos tipos: inducción completa e incompleta. La forma completa se caracteriza por las conclusiones extraídas del estudio de todas las materias de una determinada clase.

Incompleto: las conclusiones se aplican a todos los temas de la clase, pero se basan en el estudio de solo algunas unidades.

La inducción matemática completa es una inferencia basada en una conclusión general sobre la clase completa de cualquier objeto que esté relacionado funcionalmente por relaciones de una serie natural de números basada en el conocimiento de esta conexión funcional. Además, el proceso de prueba se lleva a cabo en tres etapas:

  • en el primero, se demuestra la corrección de la posición de la inducción matemática. Ejemplo: f = 1, inducción;
  • la siguiente etapa se basa en la asunción de la legalidad de la posición para todos los números naturales. Es decir, f = h, esta es una hipótesis de inducción;
  • en la tercera etapa, se demuestra la validez de la posición para el número f = h + 1, en función de la corrección de la posición del punto anterior; esta es una transición de inducción o un paso de inducción matemática. Un ejemplo es el llamado si el primer hueso de una fila cae (base), entonces todos los huesos de una fila caerán (transición).

Tanto en broma como en serio

Para facilitar la percepción, los ejemplos de resolución mediante el método de inducción matemática se denuncian en forma de problemas de broma. Esta es la tarea de Cola educada:

  • Las reglas de conducta prohíben que un hombre haga cola frente a una mujer (en tal situación, ella puede seguir adelante). Según esta afirmación, si la última persona en la cola es un hombre, el resto son hombres.

Un ejemplo sorprendente del método de inducción matemática es el problema del "vuelo sin dimensiones":

  • Se requiere demostrar que cualquier número de personas puede caber en el minibús. Es cierto que una persona puede caber dentro del vehículo sin dificultad (base). Pero no importa qué tan lleno esté el minibús, siempre cabrá 1 pasajero en él (paso de inducción).

Círculos familiares

Los ejemplos de resolución de problemas y ecuaciones mediante el método de inducción matemática son bastante comunes. Como ilustración de este enfoque, podemos considerar el siguiente problema.

Condición: h se colocan círculos en el plano. Se requiere demostrar que para cualquier disposición de las figuras, el mapa formado por ellas se puede colorear correctamente con dos colores.

Solución: para h = 1, la verdad del enunciado es obvia, por lo tanto, la prueba se construirá para el número de círculos h + 1.

Supongamos que el enunciado es válido para cualquier mapa y que se dan h + 1 círculos en el plano. Al eliminar uno de los círculos del total, puede obtener un mapa correctamente coloreado con dos colores (blanco y negro).

Al restaurar un círculo eliminado, el color de cada área cambia al opuesto (en este caso, dentro del círculo). Resulta que el mapa está correctamente coloreado en dos colores, que es lo que teníamos que demostrar.

Ejemplos con números naturales

La aplicación del método de inducción matemática se muestra claramente a continuación.

Ejemplos de soluciones:

Demuestre que para cualquier h será correcta la siguiente igualdad:

1 2 +2 2 +3 2 +… + h 2 = h (h + 1) (2h + 1) / 6.

1. Sea h = 1, lo que significa:

R 1 = 1 2 = 1 (1 + 1) (2 + 1) / 6 = 1

De esto se deduce que para h = 1 la afirmación es correcta.

2. Suponiendo que h = d, se obtiene la siguiente ecuación:

R 1 = re 2 = re (re + 1) (2d + 1) / 6 = 1

3. Suponiendo que h = d + 1, resulta:

R d + 1 = (d + 1) (d + 2) (2d + 3) / 6

R d + 1 = 1 2 +2 2 +3 2 +… + d 2 + (d + 1) 2 = d (d + 1) (2d + 1) / 6 + (d + 1) 2 = (d ( d + 1) (2d + 1) +6 (d + 1) 2) / 6 = (d + 1) (d (2d + 1) +6 (k + 1)) / 6 =

(re + 1) (2d 2 + 7d + 6) / 6 = (re + 1) (2 (re + 3/2) (re + 2)) / 6 = (re + 1) (re + 2) ( 2d + 3) / 6.

Así, se demuestra la validez de la igualdad para h = d + 1, por tanto el enunciado es verdadero para cualquier número natural, lo que se muestra en el ejemplo de la solución por inducción matemática.

Tarea

Condición: se requiere prueba de que para cualquier valor de h la expresión 7 h -1 es divisible por 6 sin residuo.

Solución:

1. Suponga que h = 1, en este caso:

R 1 = 7 1-1 = 6 (es decir, divisible por 6 sin resto)

Por tanto, para h = 1 el enunciado es verdadero;

2. Sea h = dy 7 d -1 divisible por 6 sin residuo;

3. La prueba de la validez del enunciado para h = d + 1 es la fórmula:

R re +1 = 7 d +1 -1 = 7 ∙ 7 d -7 + 6 = 7 (7 d -1) +6

En este caso, el primer término es divisible por 6 según el supuesto del primer punto, y el segundo término es 6. La afirmación de que 7 h -1 es divisible por 6 sin un resto para cualquier h natural es verdadera.

Falacia del juicio

A menudo en las demostraciones se utiliza un razonamiento incorrecto, debido a la inexactitud de las construcciones lógicas utilizadas. Esto ocurre principalmente cuando se viola la estructura y la lógica de la prueba. Un ejemplo de razonamiento incorrecto es la siguiente ilustración.

Tarea

Condición: Se requiere prueba de que cualquier montón de piedras no es un montón.

Solución:

1. Suponga que h = 1, en este caso hay 1 piedra en el montón y la afirmación es verdadera (base);

2. Suponga que para h = d es cierto que un montón de piedras no es un montón (supuesto);

3. Sea h = d + 1, de donde se sigue que cuando se agrega una piedra más, el conjunto no será un montón. La conclusión sugiere por sí misma que el supuesto es válido para todos los números naturales h.

El error radica en el hecho de que no existe una definición de cuántas piedras forman una pila. Tal omisión se denomina generalización apresurada en el método de inducción matemática. El ejemplo muestra esto claramente.

Inducción y las leyes de la lógica.

Históricamente, siempre caminan de la mano. Disciplinas científicas como la lógica, la filosofía las describen en forma de opuestos.

Desde el punto de vista de la ley de la lógica en las definiciones inductivas, existe una confianza en los hechos y la veracidad de las premisas no determina la exactitud del enunciado resultante. A menudo, las conclusiones se obtienen con cierto grado de probabilidad y plausibilidad, que, por supuesto, deben verificarse y confirmarse mediante investigaciones adicionales. Un ejemplo de inducción en lógica puede ser la declaración:

Sequía en Estonia, sequía en Letonia, sequía en Lituania.

Estonia, Letonia y Lituania son los estados bálticos. Sequía en todos los estados bálticos.

Del ejemplo, podemos concluir que no se puede obtener nueva información o verdad utilizando el método de inducción. Todo lo que se puede contar es una posible veracidad de las conclusiones. Además, la veracidad de las premisas no garantiza las mismas conclusiones. Sin embargo, este hecho no significa que la inducción vegete al margen de la deducción: un gran número de proposiciones y leyes científicas se fundamentan utilizando el método de inducción. Un ejemplo son las mismas matemáticas, biología y otras ciencias. Esto se debe principalmente al método de inducción completa, pero en algunos casos también es aplicable la inducción parcial.

La venerable era de la inducción le permitió penetrar en casi todas las esferas de la actividad humana: esto es ciencia, economía e inferencias cotidianas.

Inducción en el entorno científico

El método de inducción requiere una actitud escrupulosa, ya que demasiado depende del número de particularidades estudiadas del conjunto: cuanto mayor sea el número estudiado, más confiable será el resultado. Con base en esta característica, las leyes científicas obtenidas por el método de inducción se prueban durante mucho tiempo a nivel de supuestos probabilísticos para aislar y estudiar todos los posibles elementos estructurales, conexiones e influencias.

En ciencia, la conclusión de inducción se basa en signos significativos, con la excepción de disposiciones aleatorias. Este hecho es importante debido a la especificidad del conocimiento científico. Esto se ve claramente en los ejemplos de inducción en ciencia.

Hay dos tipos de inducción en el mundo científico (en relación con la forma de estudio):

  1. selección de inducción (o selección);
  2. inducción - exclusión (eliminación).

El primer tipo se distingue por la selección metódica (escrupulosa) de muestras de la clase (subclases) de sus diferentes áreas.

Un ejemplo de este tipo de inducción es el siguiente: la plata (o sales de plata) purifica el agua. La conclusión se basa en observaciones a largo plazo (una especie de selección de confirmaciones y refutaciones: selección).

El segundo tipo de inducción se basa en conclusiones que establecen relaciones causales y excluyen circunstancias que no corresponden a sus propiedades, a saber, universalidad, adherencia a la secuencia temporal, necesidad y unicidad.

Inducción y deducción desde una perspectiva filosófica

Si nos fijamos en la retrospectiva histórica, Sócrates mencionó por primera vez el término "inducción". Aristóteles describió ejemplos de inducción en filosofía en un diccionario terminológico más aproximado, pero la cuestión de la inducción incompleta permanece abierta. Tras la persecución del silogismo aristotélico, el método inductivo comenzó a reconocerse como fructífero y el único posible en las ciencias naturales. Bacon es considerado el padre de la inducción como un método especial independiente, pero no logró separar, como exigían sus contemporáneos, la inducción del método deductivo.

El desarrollo posterior de la inducción fue llevado a cabo por J. Mill, quien consideró la teoría de la inducción desde el punto de vista de cuatro métodos principales: concordancia, diferencia, residuos y cambios correspondientes. No es sorprendente que hoy en día los métodos enumerados, cuando se examinan en detalle, sean deductivos.

La conciencia de la inconsistencia de las teorías de Bacon y Mill llevó a los científicos a estudiar la base probabilística de la inducción. Sin embargo, incluso aquí no estuvo exento de extremos: se intentó reducir la inducción a la teoría de la probabilidad con todas las consecuencias consiguientes.

La inducción recibe un voto de confianza en la aplicación práctica en ciertas áreas temáticas y debido a la precisión métrica de la base inductiva. Un ejemplo de inducción y deducción en filosofía puede considerarse la Ley de la gravitación universal. En la fecha del descubrimiento de la ley, Newton pudo verificarla con una precisión del 4 por ciento. Y cuando se verificó más de doscientos años después, la corrección se confirmó con una precisión del 0,0001 por ciento, aunque la verificación se llevó a cabo mediante las mismas generalizaciones inductivas.

La filosofía moderna presta más atención a la deducción, que está dictada por un deseo lógico de derivar nuevos conocimientos (o verdades) de lo ya conocido, sin recurrir a la experiencia, a la intuición, pero operando con un razonamiento "puro". Al referirse a las premisas verdaderas en el método deductivo, en todos los casos, el resultado es un enunciado verdadero.

Esta característica muy importante no debe eclipsar el valor del método inductivo. Dado que la inducción, apoyándose en los logros de la experiencia, también se convierte en un medio para procesarla (incluida la generalización y la sistematización).

Aplicación de la inducción en economía

La inducción y la deducción se han utilizado durante mucho tiempo como métodos para estudiar economía y pronosticar su desarrollo.

El rango de aplicación del método de inducción es bastante amplio: el estudio del cumplimiento de los indicadores de pronóstico (ganancia, depreciación, etc.) y una evaluación general del estado de la empresa; formación de una política eficaz para la promoción de una empresa basada en hechos y sus interconexiones.

El mismo método de inducción se aplica en los gráficos de Shewhart, donde, bajo el supuesto de que los procesos se dividen en procesos controlados y no controlados, se afirma que el marco de un proceso controlado está inactivo.

Cabe señalar que las leyes científicas se fundamentan y confirman mediante el método de inducción, y dado que la economía es una ciencia que a menudo utiliza análisis matemático, teoría del riesgo y estadística, no es de extrañar que la inducción esté en la lista de métodos básicos.

Un ejemplo de inducción y deducción en economía es la siguiente situación. El aumento del precio de los alimentos (de la canasta del consumidor) y de los bienes esenciales empuja al consumidor a pensar en el alto costo emergente en el estado (inducción). Al mismo tiempo, a partir del hecho de los precios altos, utilizando métodos matemáticos, es posible derivar indicadores de aumentos de precios para bienes individuales o categorías de bienes (deducción).

Muy a menudo, el personal de gestión, los gerentes y los economistas recurren al método de inducción. Para que sea posible con suficiente veracidad predecir el desarrollo de una empresa, el comportamiento del mercado, las consecuencias de la competencia, se necesita un enfoque inductivo-deductivo para el análisis y procesamiento de la información.

Un ejemplo ilustrativo de inducción en economía relacionado con juicios erróneos:

  • el beneficio de la empresa se redujo en un 30%;
    una empresa competidora ha ampliado su línea de productos;
    nada más ha cambiado;
  • la política de producción de una empresa competidora provocó una reducción del 30% en los beneficios;
  • por lo tanto, se requiere la misma política de producción.

El ejemplo es una colorida ilustración de cómo el uso inepto de la inducción puede arruinar un negocio.

Deducción e inducción en psicología

Dado que hay un método, entonces, de acuerdo con la lógica de las cosas, también hay un pensamiento debidamente organizado (para usar el método). La psicología como ciencia que estudia los procesos mentales, su formación, desarrollo, interconexiones, interacciones, presta atención al pensamiento "deductivo", como una de las formas de manifestación de la deducción y la inducción. Desafortunadamente, en las páginas de psicología en Internet, prácticamente no hay evidencia de la integridad del método deductivo-inductivo. Aunque los psicólogos profesionales se enfrentan con mayor frecuencia a manifestaciones de inducción, o mejor dicho, a conclusiones erróneas.

Un ejemplo de inducción en psicología, como ilustración de juicios erróneos, es la afirmación: mi madre está engañando, por lo tanto, todas las mujeres son engañadoras. Se pueden extraer incluso más ejemplos "erróneos" de inducción de la vida:

  • un estudiante es incapaz de nada si ha recibido un deuce en matemáticas;
  • es un tonto;
  • el es inteligente;
  • Puedo hacer cualquier cosa;

Y muchos otros juicios de valor derivados de mensajes absolutamente aleatorios y, en ocasiones, insignificantes.

Cabe señalar: cuando la falacia de los juicios de una persona llega al punto del absurdo, hay un frente para que el psicoterapeuta funcione. Un ejemplo de inducción en una cita con un especialista:

“El paciente está absolutamente seguro de que el color rojo solo es peligroso para él en cualquier forma. Como resultado, una persona ha excluido este esquema de color de su vida, tanto como sea posible. En casa, hay muchas oportunidades para vivir cómodamente. Puede descartar todos los elementos rojos o reemplazarlos con análogos hechos en un esquema de color diferente. Pero en lugares públicos, en el trabajo, en una tienda, es imposible. Al entrar en una situación estresante, el paciente experimenta cada vez una "avalancha" de estados emocionales completamente diferentes, que pueden ser peligrosos para los demás ".

Este ejemplo de inducción, además de inconsciente, se llama "ideas fijas". Si esto le sucede a una persona mentalmente sana, podemos hablar de falta de organización de la actividad mental. La forma de deshacerse de los estados obsesivos puede ser el desarrollo elemental del pensamiento deductivo. En otros casos, los psiquiatras trabajan con estos pacientes.

Los ejemplos anteriores de inducción indican que "el desconocimiento de la ley no exime de consecuencias (juicios erróneos)".

Los psicólogos, que trabajan en el tema del pensamiento deductivo, han compilado una lista de recomendaciones diseñadas para ayudar a las personas a dominar este método.

El primer elemento es la solución de problemas. Como puede ver, esa forma de inducción, que se usa en matemáticas, puede considerarse "clásica", y el uso de este método contribuye a la "disciplina" de la mente.

La siguiente condición para el desarrollo del pensamiento deductivo es la ampliación de los propios horizontes (quien piensa con claridad, lo expresa con claridad). Esta recomendación dirige el “sufrimiento” al tesoro de las ciencias y la información (bibliotecas, sitios web, iniciativas educativas, viajes, etc.).

Por otra parte, conviene mencionar la denominada "inducción psicológica". Este término, aunque con poca frecuencia, se puede encontrar en Internet. Todas las fuentes no dan al menos una breve formulación de la definición de este término, pero se refieren a "ejemplos de la vida", mientras que presentan como un nuevo tipo de inducción, ya sea sugestión, o algunas formas de enfermedad mental, o estados extremos de la vida humana. Psique. De todo lo anterior, está claro que un intento de deducir un "nuevo término", basándose en premisas falsas (a menudo no verdaderas), condena al experimentador a recibir una declaración errónea (o apresurada).

Cabe señalar que una referencia a los experimentos de 1960 (sin especificar la ubicación, los nombres de los experimentadores, la muestra de sujetos y, lo que es más importante, el propósito del experimento) parece, por decirlo suavemente, poco convincente y la afirmación que el cerebro perciba la información sin pasar por todos los órganos de percepción (la frase "Está bajo la influencia" en este caso encajaría de manera más orgánica), te hace pensar en la credulidad y falta de crítica del autor del enunciado.

En lugar de una conclusión

La reina de las ciencias es la matemática, no en vano utiliza todas las reservas posibles del método de inducción y deducción. Los ejemplos considerados nos permiten concluir que la aplicación superficial e inepta (irreflexiva, como dicen) de los métodos más precisos y fiables siempre conduce a resultados erróneos.

En la conciencia de masas, el método de deducción está asociado con el famoso Sherlock Holmes, quien en sus construcciones lógicas a menudo usa ejemplos de inducción, usando la deducción en las situaciones adecuadas.

El artículo consideró ejemplos de la aplicación de estos métodos en diversas ciencias y esferas de la vida humana.

Método de inducción matemática

Introducción

Parte principal

  1. Inducción completa e incompleta
  2. Principio de inducción matemática
  3. Método de inducción matemática
  4. Ejemplos de soluciones
  5. Igualdad
  6. División de números
  7. Desigualdades

Conclusión

Lista de literatura usada

Introducción

Toda la investigación matemática se basa en métodos deductivos e inductivos. El método deductivo de razonamiento es el razonamiento de lo general a lo particular, es decir. razonamiento, cuyo punto de partida es el resultado general, y el punto final es el resultado particular. La inducción se usa cuando se pasa de resultados particulares a generales, es decir, es lo opuesto al método deductivo.

El método de inducción matemática se puede comparar con el progreso. Comenzamos desde lo más bajo, como resultado del pensamiento lógico llegamos a lo más alto. El hombre siempre se ha esforzado por progresar, por la capacidad de desarrollar lógicamente su pensamiento, lo que significa que la naturaleza misma pretendía que pensara inductivamente.

Aunque el campo de aplicación del método de inducción matemática ha crecido, se le dedica poco tiempo en el currículo escolar. Bueno, diles que dos o tres lecciones traerán una persona útil, para la cual escuchará cinco palabras de teoría, resolverá cinco problemas primitivos y, como resultado, recibirá una A por no saber nada.

Pero es muy importante poder pensar de forma inductiva.

Parte principal

Según su significado original, la palabra "inducción" se aplica al razonamiento con la ayuda del cual se obtienen conclusiones generales, basadas en una serie de enunciados particulares. El método más simple de razonamiento de este tipo es la inducción completa. He aquí un ejemplo de este razonamiento.

Sea necesario establecer que todo número par natural n dentro de 4< n < 20 представимо в виде суммы двух простых чисел. Для этого возьмём все такие числа и выпишем соответствующие разложения:

4=2+2; 6=3+3; 8=5+3; 10=7+3; 12=7+5;

14=7+7; 16=11+5; 18=13+5; 20=13+7.

Estas nueve igualdades muestran que cada uno de los números que nos interesan se representa de hecho como una suma de dos términos simples.

Por tanto, la inducción completa significa que el enunciado general se prueba por separado en cada uno de un número finito de casos posibles.

A veces, el resultado general se puede predecir después de considerar no todos, sino un número suficientemente grande de casos especiales (la llamada inducción incompleta).

El resultado obtenido por inducción incompleta, sin embargo, sigue siendo solo una hipótesis hasta que sea probado por un razonamiento matemático exacto que cubra todos los casos especiales. En otras palabras, la inducción incompleta en matemáticas no se considera un método legítimo de demostración rigurosa, pero es un método poderoso para descubrir nuevas verdades.

Suponga, por ejemplo, que desea encontrar la suma de los primeros n números impares consecutivos. Consideremos casos especiales:

1+3+5+7+9=25=5 2

Después de considerar estos pocos casos especiales, se sugiere la siguiente conclusión general:

1 + 3 + 5 +… + (2n-1) = n 2

aquellos. la suma de los primeros n números impares consecutivos es n 2

Por supuesto, la observación realizada todavía no puede servir como prueba de la validez de la fórmula anterior.

La inducción completa tiene un uso limitado en matemáticas. Muchos enunciados matemáticos interesantes cubren un número infinito de casos especiales, pero no podemos verificar un número infinito de casos. La inducción incompleta a menudo conduce a resultados erróneos.

En muchos casos, la salida a este tipo de dificultad es recurrir a un método especial de razonamiento llamado método de inducción matemática. Es como sigue.

Suponga que necesita demostrar la validez de un determinado enunciado para cualquier número natural n (por ejemplo, necesita demostrar que la suma de los primeros n números impares es igual an 2). La verificación directa de esta declaración para cada valor de n es imposible, ya que el conjunto de números naturales es infinito. Para probar esta afirmación, primero verifique su validez para n = 1. Entonces se demuestra que para cualquier valor natural de k, la validez del enunciado considerado para n = k implica su validez también para n = k + 1.

Entonces, la declaración se considera probada para todo n. De hecho, la afirmación es cierta para n = 1. Pero también es cierto para el siguiente número n = 1 + 1 = 2. La validez del enunciado para n = 2 implica su validez para n = 2 +

1 = 3. Esto implica la validez de la declaración para n = 4, etc. Está claro que al final llegaremos a cualquier número natural n. Por tanto, el enunciado es verdadero para cualquier n.

Resumiendo lo dicho, formulamos el siguiente principio general.

El principio de inducción matemática.

Si una oración A (n), dependiendo de un número natural n, es verdadera para n = 1 y del hecho de que es verdadera para n = k (donde k es cualquier número natural), se deduce que también es cierta para el siguiente número n = k +1, entonces el supuesto A (n) es verdadero para cualquier número natural n.

En algunos casos, es necesario probar la validez de un determinado enunciado no para todos los números naturales, sino solo para n> p, donde p es un número natural fijo. En este caso, el principio de inducción matemática se formula de la siguiente manera.

Si el enunciado А (n) es verdadero para n = p y si А (k) ÞА (k + 1) para cualquier k> p, entonces el enunciado А (n) es verdadero para cualquier n> p.

La demostración por el método de inducción matemática se lleva a cabo de la siguiente manera. Primero, la afirmación que se está probando se verifica para n = 1, es decir se establece la verdad del enunciado A (1). Esta parte de la prueba se llama base de inducción. Luego viene la parte de la demostración llamada paso de inducción. En esta parte, probamos la validez de la afirmación para n = k + 1 bajo el supuesto de que la afirmación es válida para n = k (la hipótesis de inducción), es decir, demuestre que A (k) ÞA (k + 1).

Demuestre que 1 + 3 + 5 +… + (2n-1) = n 2.

Solución: 1) Tenemos n = 1 = 1 2. Por eso,

la afirmación es verdadera para n = 1, es decir A (1) es cierto.

2) Demostremos que А (k) ÞA (k + 1).

Sea k cualquier número natural y el enunciado sea verdadero para n = k, es decir,

1 + 3 + 5 +… + (2k-1) = k 2.

Demostremos que entonces el enunciado también es cierto para el siguiente número natural n = k + 1, es decir, qué

1 + 3 + 5 +… + (2k + 1) = (k + 1) 2.

En efecto,

1 + 3 + 5 +… + (2k-1) + (2k + 1) = k 2 + 2k + 1 = (k + 1) 2.

Entonces, A (k) ÞA (k + 1). Con base en el principio de inducción matemática, concluimos que el supuesto A (n) es cierto para cualquier nÎN.

Pruebalo

1 + x + x 2 + x 3 + ... + x n = (x n + 1 -1) / (x-1), donde x¹1

Solución: 1) Para n = 1 obtenemos

1 + x = (x 2 -1) / (x-1) = (x-1) (x + 1) / (x-1) = x + 1

por tanto, para n = 1 la fórmula es correcta; A (1) es cierto.

2) Sea k cualquier número natural y sea la fórmula verdadera para n = k, es decir

1 + x + x 2 + x 3 + ... + x k = (x k + 1 -1) / (x-1).

Demostremos que entonces la igualdad

1 + x + x 2 + x 3 + ... + x k + x k + 1 = (x k + 2 -1) / (x-1).

En efecto

1 + x + x 2 + x 3 +… + x k + x k + 1 = (1 + x + x 2 + x 3 +… + x k) + x k + 1 =

= (x k + 1 -1) / (x-1) + x k + 1 = (x k + 2 -1) / (x-1).

Entonces, A (k) ÞA (k + 1). Con base en el principio de inducción matemática, concluimos que la fórmula es verdadera para cualquier número natural n.

Demuestre que el número de diagonales de un n-gon convexo es n (n-3) / 2.

Solución: 1) Para n = 3, el enunciado es

Y 3 es astuto, porque en un triángulo

 А 3 = 3 (3-3) / 2 = 0 diagonales;

A 2 A (3) es cierto.

2) Suponga que en cualquier

convexo k-gon tiene-

А 1 sy А k = k (k-3) / 2 diagonales.

А k Demostremos que entonces en el convexo

(k + 1) -número de gon

diagonales А k + 1 = (k + 1) (k-2) / 2.

Sea A 1 A 2 A 3… A k A k + 1 -convexo (k + 1) -ángulo. Dibuja una diagonal A 1 A k en ella. Para contar el número total de diagonales de este (k + 1) -gon, necesitas contar el número de diagonales en el k-gon A 1 A 2… A k, suma k-2 al número resultante, es decir se debe tener en cuenta el número de diagonales del (k + 1) -gon que sale del vértice А k + 1 y, además, la diagonal А 1 А k.

Por lo tanto,

 k + 1 =  k + (k-2) + 1 = k (k-3) / 2 + k-1 = (k + 1) (k-2) / 2.

Entonces, A (k) ÞA (k + 1). Debido al principio de inducción matemática, la afirmación es verdadera para cualquier n-gon convexo.

Demuestre que para cualquier n el siguiente enunciado es verdadero:

1 2 +2 2 +3 2 +… + n 2 = n (n + 1) (2n + 1) / 6.

Solución: 1) Sea n = 1, luego

X 1 = 1 2 = 1 (1 + 1) (2 + 1) / 6 = 1.

Por tanto, para n = 1 el enunciado es verdadero.

2) Suponga que n = k

X k = k 2 = k (k + 1) (2k + 1) / 6.

3) Considere esta afirmación para n = k + 1

X k + 1 = (k + 1) (k + 2) (2k + 3) / 6.

X k + 1 = 1 2 +2 2 +3 2 +… + k 2 + (k + 1) 2 = k (k + 1) (2k + 1) / 6 + + (k + 1) 2 = (k (k + 1) (2k + 1) +6 (k + 1) 2) / 6 = (k + 1) (k (2k + 1) +

6 (k + 1)) / 6 = (k + 1) (2k 2 + 7k + 6) / 6 = (k + 1) (2 (k + 3/2) (k +

2)) / 6 = (k + 1) (k + 2) (2k + 3) / 6.

Hemos probado la validez de la igualdad para n = k + 1, por lo tanto, en virtud del método de inducción matemática, el enunciado es verdadero para cualquier n natural.

Demuestre que para cualquier n natural la siguiente igualdad es verdadera:

1 3 +2 3 +3 3 +… + n 3 = n 2 (n + 1) 2/4.

Solución: 1) Sea n = 1.

Entonces X 1 = 1 3 = 1 2 (1 + 1) 2/4 = 1.

Vemos que para n = 1 el enunciado es verdadero.

2) Suponga que la igualdad es verdadera para n = k

X k = k 2 (k + 1) 2/4.

3) Probemos la verdad de esta afirmación para n = k + 1, es decir

X k + 1 = (k + 1) 2 (k + 2) 2/4. X k + 1 = 1 3 +2 3 +… + k 3 + (k + 1) 3 = k 2 (k + 1) 2/4 + (k + 1) 3 = (k 2 (k ++ 1) 2 +4 (k + 1) 3) / 4 = (k + 1) 2 (k 2 + 4k + 4) / 4 = (k + 1) 2 (k + 2) 2/4.

De la prueba dada, está claro que el enunciado es verdadero para n = k + 1, por lo tanto, la igualdad es verdadera para cualquier número natural n.

Pruebalo

((2 3 +1) / (2 3 -1)) ´ ((3 3 +1) / (3 3 -1)) ´… ´ ((n 3 +1) / (n 3 -1)) = 3n (n + 1) / 2 (n 2 + n + 1), donde n> 2.

Solución: 1) Para n = 2, la identidad se ve así: (2 3 +1) / (2 3 -1) = (3´2´3) / 2 (2 2 + 2 + 1),

aquellos. es verdad.

2) Suponga que la expresión es verdadera para n = k

(2 3 +1) / (2 3 -1) ´… ´ (k 3 +1) / (k 3 -1) = 3k (k + 1) / 2 (k 2 + k + 1).

3) Demostremos la exactitud de la expresión para n = k + 1.

(((2 3 +1) / (2 3 -1)) ´… ´ ((k 3 +1) / (k 3 -1))) ´ ((((k + 1) 3 +

1) / ((k + 1) 3-1)) = (3k (k + 1) / 2 (k 2 + k + 1)) ´ ((k + 2) ((k +

1) 2 - (k + 1) +1) / k ((k + 1) 2 + (k + 1) +1)) = 3 (k + 1) (k + 2) / 2´

´ ((k + 1) 2 + (k + 1) +1).

Hemos probado la igualdad y para n = k + 1, por lo tanto, por el método de inducción matemática, el enunciado es verdadero para cualquier n> 2

Pruebalo

1 3-2 3 +3 3-4 3 +… + (2n-1) 3 - (2n) 3 = -n 2 (4n + 3)

para cualquier n natural.

Solución: 1) Sea n = 1, luego

1 3 -2 3 =-1 3 (4+3); -7=-7.

2) Suponga que n = k, entonces

1 3 -2 3 +3 3-4 3 +… + (2k-1) 3 - (2k) 3 = -k 2 (4k + 3).

3) Demostremos la verdad de esta afirmación para n = k + 1

(1 3-2 3 +… + (2k-1) 3 - (2k) 3) + (2k + 1) 3 - (2k + 2) 3 = -k 2 (4k + 3) +

+ (2k + 1) 3 - (2k + 2) 3 = - (k + 1) 3 (4 (k + 1) +3).

También se demostró la validez de la igualdad para n = k + 1, por lo que el enunciado es verdadero para cualquier número natural n.

Demuestra la exactitud de la identidad.

(1 2 / 1´3) + (2 2 / 3´5) +… + (n 2 / (2n-1) ´ (2n + 1)) = n (n + 1) / 2 (2n + 1)

para cualquier n natural.

1) Para n = 1, la identidad es verdadera 1 2 / 1´3 = 1 (1 + 1) / 2 (2 + 1).

2) Suponga que para n = k

(1 2 / 1´3) +… + (k 2 / (2k-1) ´ (2k + 1)) = k (k + 1) / 2 (2k + 1).

3) Demostremos que la identidad es verdadera para n = k + 1.

(1 2 / 1´3) +… + (k 2 / (2k-1) (2k + 1)) + (k + 1) 2 / (2k + 1) (2k + 3) = (k (k + 1) / 2 (2k + 1)) + ((k + 1) 2 / (2k + 1) (2k + 3)) = ((k + 1) / (2k + 1)) ´ ((k / 2 ) + ((k + 1) / (2k + 3))) = (k + 1) (k + 2) ´ (2k + 1) / 2 (2k + 1) (2k + 3) = (k + 1 ) (k + 2) / 2 (2 (k + 1) +1).

A partir de la prueba dada, queda claro que el enunciado es verdadero para cualquier número natural n.

Demuestre que (11 n + 2 +12 2n + 1) es divisible por 133 sin residuo.

Solución: 1) Sea n = 1, luego

11 3 +12 3 = (11 + 12) (11 2 -132 + 12 2) = 23´133.

Pero (23´133) es divisible por 133 sin un residuo, por lo que para n = 1 el enunciado es verdadero; A (1) es cierto.

2) Suponga que (11 k + 2 + 12 2k + 1) es divisible por 133 sin residuo.

3) Demostremos que en este caso

(11 k + 3 +12 2k + 3) es divisible por 133 sin residuo. De hecho, 11 k + 3 +12 2n + 3 = 11´11 k + 2 +12 2´ 12 2k + 1 = 11´11 k + 2 +

+ (11 + 133) ´12 2k + 1 = 11 (11 k + 2 +12 2k + 1) + 133´12 2k + 1.

La suma resultante es divisible por 133 sin residuo, ya que su primer término es divisible por 133 sin residuo por supuesto, y en el segundo de los factores es 133. Entonces, A (k) ÞA (k + 1). En virtud del método de inducción matemática, se prueba el enunciado.

Demuestre que para cualquier n 7 n -1 es divisible por 6 sin residuo.

Solución: 1) Sea n = 1, entonces X 1 = 7 1 -1 = 6 se divide por 6 sin un residuo. Esto significa que para n = 1 la afirmación es verdadera.

2) Suponga que para n = k

7 k -1 es divisible por 6 sin residuo.

3) Demostremos que el enunciado es verdadero para n = k + 1.

X k + 1 = 7 k + 1 -1 = 7´7 k -7 + 6 = 7 (7 k -1) +6.

El primer término es divisible por 6, ya que 7 k -1 es divisible por 6 por supuesto, y el segundo término es 6. Entonces 7 n -1 es un múltiplo de 6 para cualquier n natural. En virtud del método de inducción matemática, se prueba el enunciado.

Demuestre que 3 3n-1 +2 4n-3 es divisible por 11 para una n ronda n arbitraria.
Solución: 1) Sea n = 1, luego

X 1 = 3 3 - 1 +2 4 - 3 = 3 2 +2 1 = 11 es divisible por 11 sin residuo. Por tanto, para n = 1 el enunciado es verdadero.

2) Suponga que para n = k

X k = 3 3k-1 +2 4k-3 es divisible por 11 sin residuo.

3) Demostremos que el enunciado es verdadero para n = k + 1.

X k + 1 = 3 3 (k + 1) -1 +2 4 (k + 1) -3 = 3 3k + 2 +2 4k + 1 = 3 3´ 3 3k-1 +2 4´ 2 4k-3 =

27´3 3k-1 + 16´2 4k-3 = (16 + 11) ´3 3k-1 + 16´2 4k-3 = 16´3 3k-1 +

11´3 3k-1 + 16´2 4k-3 = 16 (3 3k-1 +2 4k-3) + 11´3 3k-1.

El primer término es divisible por 11 sin un residuo, ya que 3 3k-1 +2 4k-3 es divisible por 11 por supuesto, el segundo es divisible por 11, porque uno de sus factores es 11. Entonces la suma es divisible por 11 sin remanente para cualquier n natural. En virtud del método de inducción matemática, se prueba el enunciado.

Demuestre que 11 2n -1 para un n natural arbitrario es divisible por 6 sin residuo.

Solución: 1) Sea n = 1, luego 11 2 -1 = 120 es divisible por 6 sin residuo. Esto significa que para n = 1 la afirmación es verdadera.

2) Suponga que para n = k

11 2k -1 es divisible por 6 sin resto.

11 2 (k + 1) -1 = 121´11 2k -1 = 120´11 2k + (11 2k -1).

Ambos términos son divisibles por 6 sin residuo: el primero contiene un múltiplo de 6 con 120, y el segundo es divisible por 6 sin residuo por supuesto. Esto significa que la cantidad es divisible por 6 sin un resto. La afirmación se prueba mediante el método de inducción matemática.

Demuestre que 3 3n + 3 -26n-27 para un número natural arbitrario n es divisible por 26 2 (676) sin residuo.

Solución: Primero demostremos que 3 3n + 3 -1 es divisible por 26 sin residuo.

  1. Para n = 0
  2. 3 3-1 = 26 dividido por 26

  3. Suponga que para n = k
  4. 3 3k + 3-1 es divisible por 26

  5. Demostremos que la declaración

es cierto para n = k + 1.

3 3k + 6-1 = 27´3 3k + 3 -1 = 26´3 3L + 3 + (3 3k + 3 -1) –dividido en 26

Ahora probaremos el enunciado formulado en el enunciado del problema.

1) Obviamente, para n = 1 el enunciado es verdadero

3 3+3 -26-27=676

2) Suponga que para n = k

expresión 3 3k + 3 -26k-27 es divisible por 26 2 sin resto.

3) Demostremos que el enunciado es verdadero para n = k + 1

3 3k + 6-26 (k + 1) -27 = 26 (3 3k + 3 -1) + (3 3k + 3 -26k-27).

Ambos términos son divisibles por 26 2; el primero es divisible por 26 2, porque probamos la divisibilidad entre 26 de la expresión entre paréntesis, y el segundo es divisible por la hipótesis de inducción. La afirmación se prueba mediante el método de inducción matemática.

Demuestre que si n> 2 y х> 0, entonces la desigualdad

(1 + x) n> 1 + n'x.

Solución: 1) Para n = 2, la desigualdad es válida, ya que

(1 + x) 2 = 1 + 2x + x 2> 1 + 2x.

Por tanto, A (2) es verdadera.

2) Demostremos que A (k) ÞA (k + 1) si k> 2. Suponga que A (k) es verdadera, es decir, la desigualdad

(1 + x) k> 1 + k'x. (3)

Demostremos que entonces A (k + 1) también es cierto, es decir, que la desigualdad

(1 + x) k + 1> 1+ (k + 1) ´x.

De hecho, multiplicando ambos lados de la desigualdad (3) por un número positivo 1 + x, obtenemos

(1 + x) k + 1> (1 + k´x) (1 + x).

Considere el lado derecho de la última desigualdad

fincas; tenemos

(1 + k´x) (1 + x) = 1 + (k + 1) ´x + k´x 2> 1+ (k + 1) ´x.

Como resultado, obtenemos que

(1 + x) k + 1> 1+ (k + 1) ´x.

Entonces, A (k) ÞA (k + 1). Basado en el principio de inducción matemática, se puede argumentar que la desigualdad de Bernoulli es válida para cualquier

Demuestre que la desigualdad

(1 + a + a 2) m> 1 + m´a + (m (m + 1) / 2) ´a 2 para a> 0.

Solución: 1) Para m = 1

(1 + a + a 2) 1> 1 + a + (2/2) ´a 2 ambas partes son iguales.

2) Suponga que para m = k

(1 + a + a 2) k> 1 + k´a + (k (k + 1) / 2) ´a 2

3) Demostremos que para m = k + 1 la desigualdad es verdadera

(1 + a + a 2) k + 1 = (1 + a + a 2) (1 + a + a 2) k> (1 + a + a 2) (1 + k´a +

+ (k (k + 1) / 2) ´a 2) = 1 + (k + 1) ´a + ((k (k + 1) / 2) + k + 1) ´a 2 +

+ ((k (k + 1) / 2) + k) ´a 3 + (k (k + 1) / 2) ´a 4> 1+ (k + 1) ´a +

+ ((k + 1) (k + 2) / 2) ´a 2.

Hemos probado la desigualdad para m = k + 1, por lo tanto, en virtud del método de inducción matemática, la desigualdad es válida para cualquier m natural.

Demuestre que para n> 6 la desigualdad

3 n> n´2 n + 1.

Solución: reescribimos la desigualdad como

  1. Para n = 7 tenemos
  2. 3 7/2 7 = 2187/128> 14 = 2´7

    la desigualdad es verdadera.

  3. Suponga que para n = k

3) Probemos la validez de la desigualdad para n = k + 1.

3 k + 1/2 k + 1 = (3 k / 2 k) ´ (3/2)> 2k´ (3/2) = 3k> 2 (k + 1).

Dado que k> 7, la última desigualdad es obvia.

En virtud del método de inducción matemática, la desigualdad es válida para cualquier número natural n.

Demuestre que para n> 2 la desigualdad

1+ (1/2 2) + (1/3 2) +… + (1 / n 2)<1,7-(1/n).

Solución: 1) Para n = 3, la desigualdad es verdadera

1+(1/2 2)+(1/3 2)=245/180<246/180=1,7-(1/3).

  1. Suponga que para n = k

1+ (1/2 2) + (1/3 2) +… + (1 / k 2) = 1.7- (1 / k).

3) Demostremos la validez de la

igualdad para n = k + 1

(1+ (1/2 2) +… + (1 / k 2)) + (1 / (k + 1) 2)<1,7-(1/k)+(1/(k+1) 2).

Demostremos que 1,7- (1 / k) + (1 / (k + 1) 2)<1,7-(1/k+1)Û

Û (1 / (k + 1) 2) + (1 / k + 1)<1/kÛ(k+2)/(k+1) 2 <1/kÛ

Ûk (k + 2)<(k+1) 2Û k 2 +2k

Esto último es obvio, y por lo tanto

1+ (1/2 2) + (1/3 2) +… + (1 / (k + 1) 2)<1,7-(1/k+1).

La desigualdad se prueba mediante el método de inducción matemática.

Conclusión

En particular, después de haber estudiado el método de inducción matemática, mejoré mis conocimientos en esta área de las matemáticas y también aprendí cómo resolver problemas que antes estaban fuera de mi alcance.

Básicamente, estas eran tareas lógicas y entretenidas, es decir, solo aquellos que aumentan el interés en las matemáticas como ciencia. Resolver este tipo de problemas se convierte en una actividad entretenida y puede atraer cada vez a más personas curiosas a los laberintos matemáticos. En mi opinión, esta es la base de cualquier ciencia.

Continuando con el estudio del método de inducción matemática, intentaré aprender a aplicarlo no solo en matemáticas, sino también en la resolución de problemas de física, química y la vida misma.

MATEMÁTICAS:

CONFERENCIAS, PROBLEMAS, SOLUCIONES

Libro de texto / V.G. Boltyansky, Yu.V. Sidorov, M.I.Shabunin. LLC "Popurrí" 1996.

ÁLGEBRA Y INICIO DEL ANÁLISIS

Libro de texto / I.T. Demidov, A.N. Kolmogorov, S.I.Schwarzburg, O.S.Ivashev-Musatov, B.E. Weitz. "Ilustración" 1975.